Você está na página 1de 55

Prof.

Fábio Frota

Progressão aritmética (P.A.) 2. Sequências crescente e decrescente


Consideremos a sequência (an)n ∈ E de domínio
Mentalize E. Dizemos que:
1o) a sequência é crescente se, e somente se, para
todo n ∈ E (com n > 1) tem-se: an > an–1.
1. Introdução
São comuns, na vida real, grandezas que sofrem 2o) a sequência é decrescente se, e somente se,
aumentos iguais em intervalos de tempos iguais. Por para todo n ∈ E (com n > 1) tem-se an < an–1.
exemplo, a produção de uma fábrica que aumenta de 3o) a sequência é estacionária se, e somente se,
100 unidades por mês, as economias de Eduardo que para todo n ∈ E (com n > 1) tem-se: an = an-1.
crescem todo mês de 500 dólares etc... Nesta aula,

EsPCEx – AFA – EFOMM – EEAR | MATEMÁTICA


trataremos de sequências que representam os valores Exemplos:
dessas grandezas, ou seja, de sequências (an) = (a1,
A) a sequência (2; 7; 20; 42; 70) é crescente.
a2, ... an, ...) nas quais cada termo é obtido do anterior
por um aumento constante. B) a sequência (18; 14; 12; 3; –4; –20) é
Uma progressão aritmética é uma sequência (a1, decrescente.
a2, a3, ..., an, ...) de números an, na qual é constante a C) a sequência (8; 8; 8; 8; 8) é estacionária.
diferença entre cada termo an+1 e o seu antecedente an. D) a sequência (4; 6; 17; 20; 19; 18; 2) não é
Essa diferença constante é chamada de razão e será crescente, nem decrescente, nem estacionária.
representada por r. Assim, uma progressão aritmética
de razão r é uma sequência (an) na qual an+1 – an = r,
para todo n natural. 3. Classificação das progressões
Normalmente os termos de uma progressão são aritméticas (P.A.)
numerados com índices a partir do índice 1: a1, a2, a3... . Consideremos a progressão aritmética (an)n ∈ E de
Entretanto, há casos em que é mais natural começar domínio E e razão r. Valem as propriedades:
com a0. Essa diferença de notação pode efetuar
algumas fórmulas.
1º) A P.A. é crescente ↔ r > 0
Uma progressão aritmética, de razão r, é uma
sequência tal que: an = an –1 + r(n > 1) De fato: A P.A. é crescente ↔ an > an–1 ↔
an–1 + r > an–1 ↔ r > 0.
Exemplos:
2º) A P.A. é decrescente ↔ r < 0
A) Consideremos a sequência (3; 5; 7; 9; 11).
A verificação dessa propriedade é análoga à
Vemos que cada termo, a partir do segundo, é
verificação da 1a.
igual ao anterior somado com dois. Dizemos,
então, que a sequência é uma progressão 3º) A P.A. é estacionária ↔ r = 0
aritmética de razão r = 2. A verificação dessa propriedade também é
análoga à verificação da 1a.
B) A sequência (2; 7; 12; 17; 22; 27) é uma
progressão aritmética de razão igual a 5.
C) A sequência (20; 17; 14; 11; 8; 5; 2; –1) é uma 4. Termo geral de uma P.A.
P.A de razão r = –3. Seja (an) uma P.A. com primerio termo a1 e razão
D) A sequência (5; 5; 5; 5; 5) é uma P.A. de razão r. Da definição de P.A., temos:
r = 0. a2 = a1 + r
a3 = a2 + r = a1 + r + r = a1 + 2r
4 5 7 8 
E) A sequência  , , 2; , ; 3  é uma P.A. de a4 = a3 + r = a1 + 2r + r = a1 + 3r
3 3 3 3 
a5 = a4 + r = a1 + 3r + r = a1 + 4r
1 a6 = a5 + r = a1 + 4r + r = a1 + 5r
razão r = .
3

F) Consideremos a P.A. infinita dada por:
e assim por diante.
a1  4 Estas igualdades sugerem que, numa progressão

an  an 1  2 aritmética, o termo de ordem n é igual à soma do
primeiro termo com o produto de (n – 1) pela razão, ou
A razão dessa P.A. é r = –2 e seus primeiros termos seja:
estão representados: (4; 2; 0; –2; –4; –6; ...). an = a1 + (n –1) . r, ∀ n ∈ N*

1 OS.:2196/13-Juliana
Prof. Fábio Frota

Exercícios de sala 05. (PUC-2006) Sobre as casas de um grande


tabuleiro de xadrez devem ser colocados grãos
de arroz, em quantidades que obedeçam a uma
01. (UECE-97) Seja (a1; a2; a3; ...) uma P.A. de razão
lei de formação sequencial, conforme é mostrado
2. Sabendo que a3 = 26 e ak = 164, então k vale:
na figura seguinte.
a) 71
→ → → → → → → →
b) 72


3 6 9 12 15 18 21 24
c) 73 → 48 45 42 39 36 33 30 27 →
d) 74 51 ... ... ... ... ... ... ...
EsPCEx – AFA – EFOMM – EEAR | MATEMÁTICA

... ... ... ... ... ... ... ...


02. (UFRN) O número de múltiplos de 7 entre 50 e 150 é: ... ... ... ... ... ... ... ...
a) 9 ... ... ... ... ... ... ... ...
b) 12 ... ... ... ... ... ... ... ...
c) 14 ... ... ... ... ... ... ... ?
d) 16
e) 23
A quantidade de grãos de arroz que devem ser
colocados na casa em que se encontra o ponto
03. (UFRJ-2000) Mister MM, o mágico da de interrogação é um número compreendido
matemática, apresentou-se diante de uma entre:
plateia com 50 fichas, cada uma contendo um
a) 170 e 175
número. Ele pediu a uma espectadora que
b) 175 e 180
ordenasse as fichas de forma que o número
c) 180 e 185
de cada uma, excetuando-se a primeira e a
última, fosse a média aritmética do número da d) 185 e 190
anterior com a da posterior. Mister MM solicitou e) 190 e 195
a seguir à espectadora que lhe informasse o
valor da décima sexta e da trigésima primeira
ficha, obtendo como resposta 103 e 58,
Exercícios de casa 1
respectivamente. Para delírio da plateia, mister
MM adivinhou então o valor da última ficha. 01. (Cesgranrio-RJ) A soma dos n primeiros termos
de uma sucessão é dada por Sn = n(n + 1).
Determine você também este valor.
Então, o vigésimo termo da sucessão é:
a) 420
04. (UFSCAR-2002) Uma função j é definida b) 380
c) 60
recursivamente como j(n  1)  5 . j(n)  2 . Sendo d) 40
5
e) 20
j(1) = 5, o valor de j(101) é:
a) 45
02. (UERJ-2002) Observe a sequência numérica
b) 50 abaixo.
c) 55 (0; 3; 8; 15; 24; ...)
d) 60 determine, em relação a essa sequência:
e) 65 a) seu 6o termo:
b) a expressão do termo de ordem n.

OS.:2196/13-Juliana
2
Prof. Fábio Frota

1 ap  ap 1  r ap  ap 1  r
03. Dada a função f ( x )  para x ≠ 0 e   
x( x  1)
ap 1  ap  r ap  ap 1  r
x ≠ –1. Então o valor da expressão:
 1  2ap  ap 1  ap 1
2005 .   é igual a:
 f (1)  f (2)  ...  f (2005)  ap 1  ap 1
Portanto, a 
p
a) 2002 2
b) 2003 Ou seja: cada termo a partir do segundo é a
c) 2004 média aritmética entre o termo anterior e o termo
d) 2005 posterior.

EsPCEx – AFA – EFOMM – EEAR | MATEMÁTICA


e) 2006
2. Termos equidistantes dos extremos
04. (UNB) Uma função f de domínio N (números
inteiros naturais) é dada por:  2.1. Definição
 f (0 )  5 Dois termos são chamados equidistantes dos

 4f (n)  3 extremos se o número de termos que precede um
( f (n  1))  4
, para n  N
deles é igual ao número de termos que sucede o outro.

Então o valor de f(60) é igual a: ap ,......, ap , ........, ak , ......, an



   
  
a) 48 ( p −1) termos ( n −k ) termos

b) 49
c) 50 Se ap e ak são termos equidistantes de a1 e an,
d) 51 então: p – 1 = n – k ⇒ p + k = 1 + n
e) 52
Por exemplo: dada a sequência:
05. Qual o décimo termo de uma P.A. onde a soma
dos n primeiros termos é n2? a1 a2 a3 a4 a5 a6 a7 a8
a) 15 (2; 4; 6; 8; 10; 12; 14; 16) P.A.
b) 16
c) 17
d) 19 3. Interpolação aritmética
e) 20 Interpolar (ou inserir) k meios aritméticos entre
dois números a e b significa obter uma P.A. de extremos
06. (MACK-SP) À taxa de 4% ao mês (juros simples), a1 = a e an = b, com (k + 2) termos.
R$200,00 dobram de valor ao fim de:
a) 18 meses Exemplo: interpolar seis meios aritméticos entre
b) 24 meses 4 e 39.
c) 25 meses Resolução:
d) 48 meses • Devemos ter a seguinte P.A.:
e) 50 meses ( 4 39 )
144424443
6 termos
Propriedades da progressão • Temos a1 = 4 e a8 = 39. Como k = 6, então n = k + 2,
aritmética (P.A.) Soma dos n ou seja, n = 8.
primeiros termos de uma P.A. Agora, para interpolar seis meios aritméticos,
basta determinar a razão da P.A.:
Mentalize a8 = a1 + (n – 1) . r ⇒
39 = 4 . (8 –1) . r ⇒
1. Propriedade de três termos
35 = 7r ⇒
consecutivos de uma P.A.
Numa P.A., (a1, a2, a3, ..., ap–1, ap, ap+1, ...) de r=5
razão r, temos: Portanto, a P.A. é (4, 9, 14, 19,
  24, 29, 34,
  39)
Meios interpolados

3 OS.:2196/13-Juliana
Prof. Fábio Frota

4. Representações especiais a1; a2 ; ....., ap ; ....., aq ; .....an −1; an


   
É uma frequente aparecerem problemas de ( p −1) termos ( n − q ) termos

P.A. com poucos termos. Nesses casos, pode ser útil p – 1 = n – q ou p + q = n + 1


usar representações especiais. Vamos considerar Vamos obter ap e aq:
dois casos: número ímpar de termos e número par ap = a1 + (p – 1)r
de termos.
aq = a1 + (q – 1)r
 4.1. Número ímpar de termos Somando, membro a membro, as duas
Se forem três termos, podemos representá-los por: igualdades, teremos:
x – r; x, x + r, onde r é a razão. ap + aq = 2a1 + (p + q – 2)r
Se forem cinco termos, podemos representá-los
EsPCEx – AFA – EFOMM – EEAR | MATEMÁTICA

por: x – 2r, x – r, x, x + r, x + 2r. Mas p + q = n + 1 e portanto


 4.2. Número par de termos ap + aq = 2a1 + (n – 1) . r
Se forem 4 termos, podemos representá-los por: a  (n  1)r
ap + aq = a1 + 1 
x – 3y, x – y, x + y, x + 3y, onde a razão é r = 2y. an

Se forem 6 termos: x – 5y, x – 3y, x – y, x + y, ap + aq = a1 + an


x + 3y, x + 5y.
Dica:
Num triângulo retângulo, as medidas dos lados 2. Soma dos n primeiros termos de uma
estão em P.A. P.A.
Mostre que essas medidas são proporcionais Em uma P.A., conhecendo a1, an e n a expressão
aos números 3, 4 e 5. que fornece a soma dos termos dessa P.A. é:

 4.3. Demonstração (a1  an ) . n


Sn 
2

Exercícios resolvidos
(x + r)2 = (x – r)2 + x2
2 2
01. Obter a P.A., em que a soma dos n primeiros
x 2 + 2xr + r = x2 – 2xr + r + x 2 termos é 3n2, Vn ∈ N*
x2 – 2xr – 2xr = 0
Resolução:
x2 – 4xr = 0
Se: n = 1 → S1 = a1 = 3 . 12 ∴ a1 = 3
x(x – 4r) = 0 n = 2 → S2 = a1 + a2 = 3 . 22 ∴ a1 + a2 = 12
x = 0 ou x – 4r = 0 ∴ x = 4r Como: a1 = 3 resulta:
a2 = 9, e a razão r = a2 – a1 = 6,
Soma dos n primeiros termos de uma logo (3, 9, 15, 21, ....) é a P.A.
(3, 9, 15, 21, ....)
P.A.
Neste módulo, vamos apresentar a fórmula 02. (UECE-2001.1) Considere as sequências de
que permite calcular a soma dos “n” primeiros
conjuntos: {1}, {2, 3}, {4, 5, 6}, {7, 8, 9, 10},
termos de uma P.A.
{11, 12, 13, 14, 15},...
Antes vamos apresentar uma propriedade que irá
facilitar a compreensão da fórmula da soma. Desta sequência, seja U o conjunto cujo maior
elemento é o número 91. A soma dos elementos
1. Propriedade
de U é:
A soma de dois termos equidistantes dos
extremos de uma P.A. finita é igual à soma dos a) 1100 c) 1110
extremos dessa sequência. b) 1105 d) 1115
Sabemos que ap e aq são equidistantes dos
extremos a 1 e a n se o número de termos que
Resolução:
antecedem ap é igual ao número de elementos que
Note que os últimos elementos de cada conjunto,
sucedem aq, ou seja:
pela ordem, são:

OS.:2196/13-Juliana
4
Prof. Fábio Frota

M1 = 1 portanto:
M2 = 1 + 2 = 3
M3 = 1 + 2 + 3 = 6 Resposta: B

 Nota importante
Mn = 1 + 2 + 3 + ... + n = 91 Não esqueça que:
(1 + n) . n Se a sequência (an) = (a1, a2, a3, ......., an,......) é
⇒ 2
= 91 ⇒ n2 + n – 182 = 0 ⇒
uma Progressão Aritmética, temos (Vn ∈ N*):
n  14 (não convém
 • Razão: r = an + 1 – an
ou
⇒ 

EsPCEx – AFA – EFOMM – EEAR | MATEMÁTICA


n  13
 • Termo Geral: an = a1 + (n – 1) . r

• Propriedades:
⇒ U = {79, 80, 81, 82, ...., 91}
⇒ S  (79  91) . 13  170 . 13 = 85 . 13 = 1105. 1. Se ap –1, ap e ap + 1 são três termos consecutivos
2 2 de P.A., então:
Resposta: B ap 1  ap 1
ap 
2
a+c
03. (UECE-95.2) Seja (a1, a2, a3 ... , a38, a39, a40) Nota: (...... a, b, c, .......) + b =
2
uma progressão aritmética. Se a5 + a36 = 160,
então a1 + a2 + a3 + ... + a40 é igual a: 2. Se na P.A. ap e ak são termos equidistantes dos
a) 3200 c) 6400 termos a1 e an, então:
b) 3400 d) 6800 ap + ak = a1 + an

Resolução: P.A. (...., x – R, x, x + R, ... ) 3 termos


Note que a1 + a40 = a5 + a36 = 160. Então: P.A. (..., x – 3R, x – R, x + R, x + 3R, .....)
Soma dos N primeiros termos
(a1  a40 ) . 40 160 . 40
S40    80 . 40  3200
2 2 (a1  an ) . n
Sn 
2
Resposta: A

04. (UNB-DF) Se 1+ 2 + 3 + ...... + n = 105, então o


valor de n é:
Exercícios de sala
a) 12 d) 11
b) 14 e) 15 01. (ESPM-2000) As idades de Abílio, Bernardo,
César e Dênis, em anos, formam, nessa ordem,
c) 13
uma progressão aritmética de razão 8. Se a soma
dessas idades é 100 anos, então a idade, em anos,
Resolução: de:
(1 + n) . n
1 + 2 + 3 + ..... + n =
2 a) Abílio é 15.
b) Bernardo é 20.
Então:
c) César é 32.
(1 + n) . n d) Dênis é 33.
= 105 ∴ (1 + n) . n = 210 e) Abílio é 13.
2

n2 + n – 210 = 0 02. Se a sequência (18, x, y, z, 54) é uma progressão


x+y+z
s = –1 n1 = 14 aritmética, então é igual a:
6

a) 36 d) 21
p = –210 n2 = 15 b) 42 e) 54
(não satisfaz) c) 18
5 OS.:2196/13-Juliana
Prof. Fábio Frota

03. (FATEC-97) Inserindo-se 5 números entre 18 e 07. Numa olimpíada foram colocadas, numa pista
96, de modo que a sequência (18, a2, a3, a4, a5, retilínea, trinta tochas acesas distando três
a6, 96) seja uma progressão aritmética, tem-se a3 metros uma da outra, e um recipiente contendo
igual a: água a 1 metro antes da primeira tocha. Um
corredor deveria partir do local onde está o
a) 43 recipiente, pegar a primeira tocha, retornar
b) 44 ao ponto de partida para apagá-la e repetir
c) 45 esse movimento até apagar a trigésima tocha.
d) 46 Se x expressa a quantidade total de metros
percorridos pelo corredor, então a soma dos
e) 47
algarismos que compõem o número x é igual a:
EsPCEx – AFA – EFOMM – EEAR | MATEMÁTICA

a) 15
04. (UNICAMP-SP) Os lados de um triângulo retângulo
b) 16
estão em progressão aritmética (P.A.). Sabendo-se
c) 17
que o perímetro mede 57cm, podemos afirmar que
o maior cateto mede: d) 18
e) 19
a) 17cm
b) 19cm
08. A soma de todos os números naturais de dois
c) 20cm algarismos que, divididos por 5, dão resto 2 é:
d) 23cm a) 981
e) 27cm b) 967
c) 1862
05. (IME-RJ) A relação que deve existir entre os d) 1924
números m, n, p e q para que se verifique a seguinte e) 987
igualdade entre os termos da mesma progressão
aritmética (P.A.): am + an = ap + aq, é igual a:
a) m + n + p + q = 0 Exercícios de casa 2
b) m = n + p + q
c) m + n = p + q 01. (MACK-96) As medidas dos ângulos assinalados
d) m – n = p – q na figura abaixo formam uma progressão
e) n. d. a. aritmética. Então, necessariamente, um deles
sempre mede:
06. (EPCAR-99) A figura abaixo mostra uma construção
feita com palitos de fósforo:

a) 108 d) 86
b) 104 e) 72
c) 100

O número de palitos necessários para a construção


02. (UFC) Três números em progressão aritmética
da 1a até a 10a fila será:
são tais que a sua soma é 21 e o seu produto é
a) 40. 231. Se r é a razão desta P.A., determine |r|.
b) 108. a) 4 d) 5
c) 144. b) 2 e) 1
d) 210. c) 3

OS.:2196/13-Juliana
6
Prof. Fábio Frota

03. Os três termos de uma sequência são diretamente 08. (Fuvest-SP) Em uma P.A. de termos positivos, os
proporcionais aos números 2, 5 e 7. Subtraindo 4 três primeiros termos são: 1 – a; –a; 11− a . O quarto
do termo do meio, os números passam a formar termo desta P.A. é igual a:
uma P.A. Então, o maior termo da sequência
original é igual a: a) 1
b) 2
a) 36
c) 3
b) 48
d) 4
c) 56
e) 5
d) 63
e) 72

EsPCEx – AFA – EFOMM – EEAR | MATEMÁTICA


09. (UECE) As medidas, em graus, dos ângulos
internos de um triângulo formam uma progressão
04. (FUVEST-2000) Sejam a, b, c três números aritmética e um dos ângulos mede 30o nestas
estritamente positivos em progressão aritmética. condições, a medida, em graus, do maior ângulo
Se a área do triângulo ABC, cujos vértices do triângulo é igual a:
são A = (–a, 0), B = (0, b) e C = (c, 0), é igual a b,
então o valor de b é: a) 80
b) 85
a) 5
c) 90
b) 4
d) 95
c) 3
d) 2
10. (IME-Adaptada) Se os números (logkx , logmx , lognx )
e) 1
com x, k, m e n positivos e diferentes de 1, formam
uma progressão aritmética (P.A.). Então, o valor
05. Os lados de um triângulo retângulo estão em
log m
progressão aritmética. Seja x a tangente do menor de (kn) k é igual a:
ângulo interno desse triângulo, então o calor da a) 4
n
8.x b) n2
expressão é:
3 3
n
c)
a) 1 d) 4
d) n3
b) 2 e) 5
e) n
c) 3

11. (UFC-2003) A soma dos 15 primeiros termos de


06. (UEE-94) Uma progressão aritmética de n termos
uma progressão aritmética é 150. O 8o termo desta
tem razão igual a 3. Se retirarmos os termos de
ordem ímpar, os de ordem par formarão uma P.A. é:
progressão: a) 10
a) aritmética de razão 2. b) 15
b) aritmética de razão 6. c) 20
c) aritmética de razão 9. d) 25
d) geométrica de razão 3. e) 30
e) geométrica de razão 6.

12. (F.C.M. STA. CASA) A sucessão S dos números


07. (UFC-Modificado) Se os números 9, b e c são 1, 5, 13, 25, ak, ..., possui a propriedade de que
as medidas, em cm, dos lados de um triângulo
as diferenças d k = a k + 1 – a k, como k = 1, 2, 3,
retângulo e 9, b e c formam, nesta ordem, uma
... formam uma progressão aritmética. O 30o termo
progressão aritmética (P.A.) crescente. Então, a
da sucessão S é:
medida da área do triângulo, em cm, é igual a:
a) 18 a) 120
b) 36 b) 117
c) 54 c) 871
d) 64 d) 1.741
e) 72 e) impossível de ser calculado
7 OS.:2196/13-Juliana
Prof. Fábio Frota

13. (UFC) Os números inteiros positivos são c) Uma P.G é constante se, e somente se:
agrupados em partes distintas disjuntas, da • a1 = 0, exemplo (0, 0, 0, …)
seguinte maneira: {1}, {2, 3}, {4, 5, 6}, {7, 8, 9, 10}, {11, • q = 1, exemplo (7, 7, 7, …)
12, 13, 14, 15}, ... seja S a soma dos elementos
que compõem o 24o conjunto desta sequência. d) Uma P.G é alternada quando possui sua
Calcule a soma do algarismo de S. • razão negativa, exemplo (1, –2, 4, –8, 16, …)

14. (MACK-SP) Se e) Uma P.G é estacionária quando a1 ≠ 0 e q = 0,


exemplo (7, 0, 0, 0, …)
A = 12 – 22 + 32 – 42 + 52 – 62 + ... + 992 – 1002,
A
EsPCEx – AFA – EFOMM – EEAR | MATEMÁTICA

então é igual a: 3. Notações especiais


10
a) – 505 Uma P.G. pode ser representada por:
b) – 5050
c) 505 a) (… x , x , x, x . q, x . q2, …) quando possui um
2
d) 5050 q q
e) – 100 número ímpar de termos.
x x
15. (AFA-99) Se a soma dos 6 primeiros termos de
b) (… , , x . k, x . k3, …) quando possui um
k3 k
uma progressão aritmética é 21 e o sétimo termo número par de termos. Neste caso é importante
é o triplo da soma do terceiro com o quarto termo, observar que a razão q é igual a k2.
então o primeiro termo dessa progressão é:
a) –7
4. Fórmula do termo geral
b) –8
Na P.G. (a1, a2, a3, …, an, …) tem-se que:
c) –9
a2 = a1 . q
d) – 10 a3 = a1 . q2
a4 = a1 . q3
  

Progressão geométrica (P.G.) an = a1 . qn – 1

Note que um termo a k qualquer, pode ser


Mentalize representado por ak = a1 . qk – 1. Então:

1. Definição an qn 1
 ⇒ an = ak . qn – k
Progressão geométrica (P.G.) é toda sequência ak qk 1
em que, cada termo, a partir do segundo, é igual
ao termo anterior multiplicado por uma constante q,
denominada razão. 5. Interpolação geométrica
Uma P.G. também pode ser definida pela lei de Inserir ou interpolar k meios geométricos entre a
recorrência: e b significa determinar uma P.G. em que a1 = a, an = b
a1 = k e n = k + 2.
an = an –1 . q, n ≥ 2
6. Propriedades
2. Classificação a) Se a, b e c são três termos, não nulos,
a) Uma P.G. é crescente se, e somente se: consecutivos de uma P.G., tem-se que:

• a1 > 0 e q > 1, exemplo (1, 2, 4, 8, 16, …)


b2 = a . c
1 1
• a1 < 0 e 0 < q < 1, exemplo (–3, –1, − , − , …)
3 9
b) Em uma progressão geométrica qualquer,
b) Uma P.G é decrescente se, e somente se: o produto de dois termos equidistantes dos
• a1 > 0 e 0 < q < 1, exemplo (16, 8, 4, 2, …) extremos é constante e igual ao produto dos
• a1 < 0 e q > 1, exemplo (–3, –6, –12, …) extremos, isto é:

OS.:2196/13-Juliana
8
Prof. Fábio Frota

Se P.G. (a1, a2, a3, …, an – 2, an – 1, an), então Resolução:


tem-se que: Base = b
a1 . an = a2 . an – 1 = a3 . an – 2 = … Altura = h
b.h
Área =
2
Exercícios resolvidos  b . h
 b; h; 2  P.G. razão = 8
 
01. (U. CAXIAS DO SUL/RS) O valor de x para que a b.h
sequência (x + 1, x , x + 2) seja uma progressão 2
Razão =
geométrica é: h
1 1

EsPCEx – AFA – EFOMM – EEAR | MATEMÁTICA


a) d) – 1
2 2
2
b. h 1 b
Razão = x Razão =
b) e) 3 2 h 2
3
2 8 b
c) – = ⇒ b = 16
3 1 2

Resolução:
04. (PUC/SP) Numa progressão geométrica, a
(x + 1, x , x + 2) P.G.
diferença entre o 2o e o 1o termo é 9 e a diferença
x2 = ( x + 1) . (x + 2) entre o 5 o e o 4 o termos é 576. O termo da
x2 = x2 + 2x + x + 2 progressão é:
2
x 2 = x 2 + 3x + 2 ⇒ 3x = –2 ⇒ x = –
3 a) 3
b) 4
02. (ULBRA/RS) Numa P.G. de razão 3, e primeiro c) 6
termo 8, o termo que vale 648 é o:
d) 8
a) quarto d) sétimo e) 9
b) quinto e) oitavo
c) sexto
Resolução:
a2 – a1 = 9
Resolução:
q=3 a5 – a4 = 576
a1 = 8
an = 648 a1 . q  a1  9
 4 3
n=? a1 . q  a1 . q  576

an = a1 . qn – 1 a1 ( q  1)  9
648 = 8 . 3n –1  3
()
a1 . q (q  1)  576
81 = 3n–1
4 n 1
3  3 q3 = 64 → q = 4
n–1=4
n=5 a1 . (q – 1) = 9
a1 . 3 = 9
03. (F.G.V/SP) Em um triângulo, a medida da base, a a1 = 3
medida da altura e a área formam, nessa ordem,
uma P.G. de razão 8. Então, a medida da base 05. (CESGRANRIO) Se x e y são positivos e se
vale: (x, xy, 3x) estão, nesta ordem, em progressão
geométrica, então o valor de y é:
a) 1
a) 2
b) 2
b) 2
c) 4
c) 3
d) 8
d) 3
e) 16
e) 9
9 OS.:2196/13-Juliana
Prof. Fábio Frota

Resolução: Exercícios de sala


(x, xy, 3x) P.G.
(xy)2 = x . 3x
01. (UFRN-2003) A sequência de figuras abaixo
x 2 y2 = 3 x 2
representa os cinco primeiros passos da
construção do conjunto de Sierpinski. Os vértices
y2 = 3 ⇒ y= 3
dos triângulos brancos construídos são os pontos
médios dos lados dos triângulos escuros da
06. (UFPR) Se log e2x – 1, log e2x + 1 e log e5x + 4, figura anterior. Denominamos a1, a2, a3, a4 e a5,
são termos em progressão geométrica, então os respectivamente, as áreas das regiões escuras da
valores de x são: primeira, segunda, terceira, quarta e quinta figuras
EsPCEx – AFA – EFOMM – EEAR | MATEMÁTICA

a) 0 e 1 d) 1 e 3 da sequência.
5
b) 1 e − e) 1 e –1
6
c) 1 e 2

Resolução:
(log e2x – 1, log e2x + 1 e log e5x + 4)
(log e2x + 1)2 = log e2x – 1 . log e5x + 4 ⇒
[(2x + 1) log e]2 = (2x – 1) . log e . (5x + 4) . log e
2 2
⇒ (2x + 1)2 . log e = (2x – 1) . (5x + 4) log e ⇒
Podemos afirmar que a1, a2, a3, a4 e a5 estão,
4x2 + 4x + 1 = 10x2 + 8x – 5x – 4 ⇒ nessa ordem, em progressão geométrica de razão:
6x2 – x – 5 = 0
3 1
∆ = b2 – 4ac a) c)
4 3
∆ = 1 – 4 . 6 .(–5) 1 1
∆ = 1 + 120 b) 2 d)
4
∆ = 121
02. (UECE) Seja (t1, t2, t3, t4, t5) uma progressão
geométrica de termos positivos. Se t1 . t2 . t3 . t4 .
x1 = 1
t3  4
t5 = 610, então é igual a:
1 ± 11 t3  4
x=
12
5
a)
x2 = – 5 4
6
3
b) 2

07. (FUVEST/SP) O quinto e o sétimo termos de uma 7


c)
P.G. de razão positiva valem, respectivamente, 4
10 e 16. O sexto termo desta P. G. é: d) 2
a) 13
b) 10 6 03. (UNIFOR)
c) 4
d) 4 10
e) 40

Resolução:
(a52 ; a6 ; a7 ) P.G.
a 62 A partir do segundo, os vértices de cada
= a5 . a7 ⇒
quadrado são os pontos médios dos lados do
a6 = 10 . 16 ⇒ quadrado anterior. Às medidas dos lados desses
a6 = 4 10 quadrados constituem uma progressão:
OS.:2196/13-Juliana
10
Prof. Fábio Frota

7 04. (FUVEST-SP) No primeiro dia do mês, um frasco


a) aritmética de razão recebe 3 gotas de um remédio. No segundo dia
4
ele recebe 9 gotas, no terceiro dia ele recebe 27
2 gotas, e assim por diante. No dia em que recebeu
b) aritmética de razão
2 2.187 gotas ficou completamente cheio. Em que
2 dia do mês isso aconteceu?
c) geométrica de razão
2
a) 6 d) 9
1
d) P.A. de razão 5 b) 7 e) 10
c) 8

EsPCEx – AFA – EFOMM – EEAR | MATEMÁTICA


04. ( U N E S P / S P ) Q u a n t o s t e r m o s d e v e m 05. (PUC/SP) Numa progressão geométrica de
1 termos positivos, o primeiro termo é igual à razão
s e r i n s e r i d o s e n t r e o s n ú m e r o s− e
9 e o segundo termo é 3. Qual é o oitavo termo da
81 para se ter uma P.G. de razão 3? progressão?
a) 7 d) 10
a) 81
b) 5 e) 9
c) 8 b) 37

c) 27 3
05. (UFPI) O primeiro e o segundo termo de uma P.G.,
d) 273
são respectivamente, m –4 e m t, m > 0 e m ≠ 1.
Se o 8o termo da P.G. é m52, o valor de t é: e) 333
a) 1 d) 4
b) 2 e) 5 06. (UNIFOR) Seja (a1, a2, a3, ...) uma P.G. tal que a3 .
c) 3 a4 . a5 . a6 . a7 =32. O 5o termo dessa P.G. é:
a) 4
b) 2
Exercícios de casa 3 c) 8
d) 1
01. (UFC) Se a, b e c são, respectivamente, os termos e) 16
de ordem 3, 5 e 11 de uma P.G., qual é a ordem
a. c 07. (UESPI-1999) Numa progressão geométrica de
do termo ?
b cinco termos, a soma do segundo com o quarto
a) 4 d) 9 termo é 30, e a soma do terceiro com o quinto é
b) 6 e) 12 60. Qual é o terceiro termo dessa progressão?
c) 7 a) 8
8 b) 24
02. (UECE) Numa P.G. crescente, a 4 – a 2 = e c) 6
3
4 d) 48
a2 + a3 = . A razão dessa P.G. é:
3 e) 12

a) 3 d) 6
b) 4 e) 7 Soma e produto de uma P.G. finita /
c) 5 Soma de uma P.G. infinita e P.A.G.
03. (FAESP-F.G.V.) Em um triângulo, a medida da
base, a medida da altura e a medida da área
Mentalize
formam, nesta ordem, uma P.G. de razão 8.
Então, a medida da base vale: 1. Produto dos termos
a) 1 d) 8 O produto dos n primeiros termos de uma P.G.
b) 2 e) 16 (a1, a2, a3, …, an) é dado por:
c) 4
11 OS.:2196/13-Juliana
Prof. Fábio Frota

Pn = a1 . a1 . q . a1 . q2 . … a1 . qn – 1, C) S = 1 + 4 + 16 + 64 . (4)
4S = 4 + 16 + 64 + 256
 Pn  a1n . q(1 2  3  ...  n 1) 4S – S = –1 + 256
n . ( n 1)
3S = 255
 Pn  a1n . q 2

255
 Nota importante
3
O produto dos n primeiros termos de uma P.G
(a1, a2, a3, …, an) também pode ser calculado, usando então:
o seguinte método: S = 85
EsPCEx – AFA – EFOMM – EEAR | MATEMÁTICA

Pn = a1 . a2 . a3 ...... . an – 2 . an – 1 . an,
⇒ Pn = an . an – 1 . an – 2 . … a3 . a2 . a1 Exemplo:
2 Calcule o valor de n sabendo que
⇒ Pn = a1 . an . a2 . an – 1 . a3 . an – 2 . … an . a1,
1 + 2 + 22 + ... + 2n = 127
2
P = a1 . an . a1 . an . a1 . an ... a1 . an
n
  
" n vezes " Solução:
1 + 2 + 22 + ... + 2n = 127 . (2)
⇒ Pn = (a1 . an )n/2
2 + 22 + ... + 2n + 2n + 1 = 254
É importante observar que o sinal do produto –1 + 2n + 1 = 127
é dado pela quantidade de termos negativos da P.G, Portanto
isto é: se o número de termos da P.G. é ímpar, então 2n + 1 = 128
o sinal do produto é negativo. Caso contrário, o sinal
2n + 1 = 27
do produto é positivo.
n+1=7
2. Soma dos termos de uma P.G. finita n=6
Dada a P.G. (a1, a2, a3, …, an) em que a 1 ≠ 0
e q ≠ 1. Então a soma dos seus termos é:  Nota importante
a) Se a1 = 0, exemplo (0, 0, 0, …, 0), então a
Sn = a1 + a1q + a1q2
+ .......... + a1 . qn – 1, soma dos termos é Sn = 0.
⇒ q . Sn = a1 . q + a1 . q2 + .......... + a1 . qn, b) Se q = 1, exemplo (7, 7, 7, …, 7), então
⇒ q . Sn – Sn = a1 . qn – a1 Sn = 7 . n, onde n é o número de termos.
⇒ (q – 1) . Sn = a1 . (qn – 1) c) Se a1 ≠ 0 e q = 0, exemplo (7, 0, 0, …, 0), a
soma Sn = a1 = 7.
a1 . (qn  1) an . q  q1
SSn
⇒ n=

q 1
ou 3n 
q 1
Exercícios resolvidos
1 1

. 2 . 2
2 4
Caro aluno, vamos raciocinar algumas somas de 01. (Unifor-97.1) O produto P = 22 . 21
P.G. finita de maneira diferente: .... é:
Por exemplo:
a) impossível de ser calculado.
Calcule
b) 8
A) S = 2 + 4 + 8 + 16 . (2) c)
4
8. 2
2S = 4 + 8 + 16 + 32 d) 8. 2
2S – S = –2 + 32 e) 16
Com isso:
Resolução:
S = 30 1 1 1 1
( 2 1    ...)

B) S = 1 + 3 + 9 + 27 + 81 . (3)
P  22 . 21 . 2 2 . 2 4 ...  2 2 4

1 1 1 2
3S = 3 + 9 + 27 + 81 + 24 Mas, 2  1    ...   4
2 4 1 1
3S – S = –1 + 243 1
2 2
2S = 242 Logo, P = 24 = 16
S = 121 Resposta: E
OS.:2196/13-Juliana
12
Prof. Fábio Frota

02. (ACAFE/SC) A soma dos oito primeiros termos da Resolução:


P.G. (16, 8, 4, ...) é:
(a1, a2, a3, a4) P.G. a4 = ?
255 255
a) 256 d) a2  a4  10
16

255 255 a1  a3  5
b) − e) −
256 16
a1q  a1 . q3  10
255 
c) 2
a1  a1 . q  5
8

EsPCEx – AFA – EFOMM – EEAR | MATEMÁTICA


Resolução: a1q (1  q2 )  10
a2  ()
8 1 2
(16, 8, 4, ...) P.G. =
q = = a1 (1  q )  5
a1 16 2
a1 . (qn  1) a1 . (q8  1) q=2
Sn   S8  
q 1 q 1
a1 (1 + q2) = 5 a4 = a1 . q3
  1 8 
16    1  1 
16   1 a1 . (1 + 4) = 5 a4 = 1 . 23
  2    256  
 S8   S8 
1 1 a1 = 1 a4 = 8
1 
 1 2  255 2
 S8    16  . ( 2) 
 16  8

Soma de uma P.G. infinita e P.A.G.


03. (F.C.M. Santos/SP) Em uma P.G., a 8 = 324
e q = 2. Determine a soma dos nove primeiros 1. Limite da soma dos termos de uma P.G.
termos. infinita e convergente
Diz-se que uma P.G. infinita converge se, e
a) 2000
somente se –1 < q < 1. Neste caso, é possível calcular
b) 1553
o limite da soma dos infinitos termos dessa P.G.
c) 1615
Note que, na P.G. (a1 . a1 . q, a1q2, …, a1qn, …) com
d) 1213
–1 < q < 1, quando n tende para infinito, qn tende para
e) n .d. a
zero, isto é: lim n
qn = 0.

Resolução: Caro aluno, vamos raciocinar com relação a


a8 = 324; q = 2 alguns cálculos de potenciação.
* a8 = a1 . q7 → 324 = a1 . 27 → 324 = a1 . 128 → Por exemplo:
324 Calcule:
→ a1 =
128 n
a . (q9  1) 324  1  , quando n = +∞.
*S 9  1  S9  . (29  1)   10 
q 1 128  
324 81 Solução:
 S9  . 511  S9  . 511 
128 32 Caro aluno, vamos pensar:
• P / n = 1, temos:
41.391  1
1
 S9 
32  10   0,1
 
• p / n = 2, temos:
04. (Mackenzie/SP) Numa P.G. de quatro termos, a
2
soma dos termos de ordem par é 10 e a soma dos  1
termos de ordem ímpar é 5. Então, o quarto termo  10   0, 01
 
dessa progressão vale:
• p / n = 3, temos:
a) 9 d) 15 3
b) 8 e) 10  1
 10   0, 001
c) 6  
13 OS.:2196/13-Juliana
Prof. Fábio Frota

• p / n = 20, temos:
Exercícios resolvidos
20
 1
 10   0, 00....001 01. Calcule o valor das dízimas periódicas:
 
A) 0,333....

• p / n = 1000, temos Resolução:


Caro aluno, vamos resolver esta dízima periódica
1000
 1 por uma soma de uma P.G. infinita.
 10   0, 000..............01
  0,333... = 0,3 + 0,03 + 0,003 + ...
EsPCEx – AFA – EFOMM – EEAR | MATEMÁTICA

Então, concluímos que p / n = +∞, Com isso:


a1 = 0,3 e q = 0,1

 1
 10   0 ( tende há ser zero)
  Portanto:
a
Sn  1
1 q
Observação
0, 3
Sn 
1  0,1
1
A base da potência q = (– 1 < q < 1) 0, 3
10 Sn 
0, 9
somente nesse caso.
1
Sn 
Assim sendo, quando n tende para infinito, 3
Sn tende para S, isto é:
B) 0,12525....
a . (qn  1) a . (0  1)
lim Sn  1 S 1 
n q 1 q 1
Resolução:
a1 a
S  S 1
q 1 1 q
0,12525 ........... = 0,1 +

2. Progressão aritmético-geométrico 0
, 025
 + 0
, 0025
 +
...
(P.A.G.) P.G. inf inita

Uma sequência do tipo: (a; (a + r)q; (a + 2r) q 2; a1 = 0,025


(a + 3r) q 3; ...) é chamada progressão aritmético- q = 0,01
-geométrica.
Os números a, a + r, a + 2r; ... formam uma P.A. Com isso:
de razão r e os números q, q2, q3, ... formam uma P.G. a1
Sn 
de razão q. Para acharmos a soma Sn dos n primeiros 1 q
termos dessa progressão, usamos um procedimento 0, 025
Sn 
análogo ao usado na dedução da fórmula da soma dos 1  0, 01
n primeiros termos da P.G. 0, 025
Sn 
“Multiplicamos Sn por q e fazemos a diferença 0, 99
Sn – qSn” 25
Sn 
990
Por exemplo:
A) (1; 2x; 3x2; 4x3; ... ) P.A.G. Portanto:
1 25
1 2 3 4  0,12525..... = +
B)  ; ; ; ; ...  P.A.G. 10 990
 2 4 8 16 
99 + 25
0,12525.... =
C) (1 . 3; 3 . 32; 5 . 33; ...) P.A.G. 990
124
D) (2 . 5; 4 . 52; 6 . 53; ...) P.A.G. 0,12525... =
990
OS.:2196/13-Juliana
14
Prof. Fábio Frota

02. Calcular a soma dos termos da P.G.  9 9 


 1 1  Com  = 3cm, obteremos a P.G.  9, , , ...  ,
.  2 4 
 2, 1, 2 , 4 , ...  1
  sendo a1 = 9 e q = .
2
Resolução:
1
Temos: a1 = 2, q = 2 Substituindo esses valores na expressão
a1
A soma dos termos dessa P.G. infinita é: S , temos:
1 q
a 2 2
S 1 S S  S4 9 9
aq 1 1 S S  S  18
1 1 1
1

EsPCEx – AFA – EFOMM – EEAR | MATEMÁTICA


2 2
2 2
03. Calcular a geratriz da dízima 0,232323... Portanto, a soma dos perímetros de todos os
Resolução: triângulos equiláteros é 18cm.
A dízima 0,232323... pode ser escrita como 0,23
+ 0,0023 + 0,000023 + ... .
Exercícios de sala
Daí: 01. (UFC) Se S3 = 21 e S4 = 45 são, respectivamente,
23 23 23 as somas dos três primeiros e quatro primeiros
0, 232323...     ...
100 10.000 1.000.000 termos de uma P.G., cujo termo inicial é 3,
então a soma dos cinco primeiros termos da
progressão é:
A expressão que acabamos de obter é a soma
dos termos de uma P.G. infinita decrescente, a) 66
23 1 b) 69
em que a1 = eq= .
100 100 c) 93
d) 96
Então, temos:
23 23 e) 105
a1 100 100 23
Sn   Sn   Sn   Sn 
1 q 1 99 99
1 02. (UECE) Se p é o produto dos 20 primeiros termos
100 100
de uma progressão geométrica, cujo primeiro
04. O lado de um triângulo equilátero mede termo e a razão são iguais a 7, então o valor de
3cm. Unindo-se os pontos médios de seus log7p é:
lados, obtém-se um novo triângulo equilátero.
a) 205
Unindo-se os pontos médios do novo triângulo,
obtém-se outro triângulo equilátero, e assim b) 210
sucessivamente. Determine a soma dos c) 215
perímetros de todos os triângulos. d) 220

Resolução: 03. De acordo com a disposição dos números abaixo,


2
4 6
8 10 12
16 18 20 22
32 34 36 38 40
... ... ... ... ...
a soma dos elementos da décima linha vale:
a) 10330
b) 10333
  c) 1042
Temos: S1 = 3, S2 = 3 . , S3 = 3 . , e assim
2 4 d) 1024
sucessivamente.
e) 20660
15 OS.:2196/13-Juliana
Prof. Fábio Frota

04. (UFPE) Sabendo-se que numa progressão 08. (MACK/SP) Sendo S = 1 + 2x + 3x2 + 4x3 + .....
geométrica o 1o termo é 1 e o 6o termo é 32, (0 < x < 1), pode-se afirmar que:
assinale a alternativa que corresponde ao produto 1
dos 6 primeiros termos desta progressão: a) S 
(1  x )2
a) 4096 2
b) S
b) 1024 (1  x )2
c) 5120 2
c) S
(2  x )2
d) 32768
1
e) 10000 d) S  (2  x )2
EsPCEx – AFA – EFOMM – EEAR | MATEMÁTICA

x
05. (UFC) Se log3(a2 – a1) = log12(a4 – a3) = 2, então e) S 
(2  x )2
a soma dos cinco primeiros termos da progressão
geométrica, de razão positiva, a1, a2, a3, a4, ... é
igual a:
09. (CEFET-CE) O valor da expressão:
a) 1061
b) 1023 x y x y x y... é igual a:
c) 1024
d) 1060
a) x.y
 1 1 1 
06. (ITA/SP) Dada a P.G.  1, , , ,...  , a soma b) 3
x2 . y
 2 4 8 
dos seus infinitos termos é: c) 3
xy 2
1
a)
8
d) 3 xy
b) 2
1 e) 4 x.y
c) 1 +
2n
3
d)
2 Exercícios de casa 4
e) 3

01. (UFC-Modificada) Numa progressão geométrica, o


07. (OBM) Um micróbio, de tamanho desprezível, primeiro termo é 3x e a razão é 3. Se a soma dos
parte da origem de um sistema de coordenadas. 5 primeiros termos é 1089, o valor de x é:
Inicialmente, ele se desloca uma unidade e
chega no ponto (1, 0). Aí ele vira 90º no sentido a) 1
1 b) 2
anti-horário e anda unidade até o ponto
2 c) 3
 1 d) 4
 1, 2  . Ele continua dessa maneira, sempre
  e) 5
descrevendo ângulos de 90 o no sentido anti-
-horário e andando a metade da distância da 02. (UFC) Meia-vida de uma substância radioativa é o
vez anterior. Continuando indefinidamente, tempo necessário para que sua massa se reduza
ele vai se aproximar cada vez mais de um à metade. Tomemos, hoje, 16 gramas de uma
determinado ponto. Quais são as coordenadas substância radioativa cuja meia-vida é de 5 anos.
desse ponto? (PG). Se daqui a n anos sua massa for 2–111 gramas, o
2 4 valor de n é igual a:
 1
a)  ,  c)  1, 2 
5 5   a) 525
b) 550
1  4 2 c) 565
b)  , 1 d)  , 
2  5 5 d) 575

OS.:2196/13-Juliana
16
Prof. Fábio Frota

03. (UFMG-2004) A população de uma colônia da 08. (Santa Casa/SP) Simplificando-se a expressão
bactéria E. coli dobra a cada 20 minutos. Em um 3 3 3 3 3
A=3x. x . x ... obtém-se:
experimento, colocou-se, inicialmente, em um tubo
de ensaio, uma amostra com 1 000 bactérias por
a) x d) 1
mililitros. No final do experimento, obteve-se um
3 1
total de 4.096 x 106 bactérias por mililitro. Assim b) x e)
729
sendo, o tempo do experimento foi de: 1
c)
a) 3 horas e 40 minutos. 4
b) 3 horas.
09. (EAESP-FGV/SP) Sabendo-se que 0 < q < 1,
c) 3 horas e 20 minutos. então q + 2q2 + 3q3 + 4q4 + ... é igual a:

EsPCEx – AFA – EFOMM – EEAR | MATEMÁTICA


d) 4 horas.
 q 
2 q
a)   d) 1− q
04. (UFV-2002) Se a soma dos n primeiros termos de  1 q 
n q q2
 1 b) e)
uma P.G. é dada por Sn  1    , onde n ≥ 1, 1 − q2 1− q
2
q
então o nono termo desta P.G. é: c)
1  q
2

a) 2–8 d) 28
b) 2–10 e) 29
3
c) 2–9 10. (UNIFOR) Seja G = π . 9 π . 27 π ... . O valor de
G5 é:
05. (MACK-99) Seja a sequência geométrica, de n
a) π
termos positivos, que se obtém inserindo-se k
1 b) π
meios geométricos entre 2 e 8. Se o produto de
π
todos os termos é 32, então n vale: c)
2
a) 5 d) 8 d) 1
b) 6 e) 9
c) 7
11. (CESCEM/SP) A soma da série

06. (Cesgranrio-99) Considere uma P.G. de 5 termos 1 1 1 1 1 1 1 1


+ + + + ... + n + n + n +1 + n +1 + ... é:
e razão positiva, onde a soma do primeiro com o 2 3 4 9 2 3 2 3
9
terceiro termo é e o produto de seus termos é 5
2 a)
3
1024. O produto dos três termos iniciais dessa
b) 1
progressão é igual a:
1 3
c)
a) d) 4 2 2
2
b) 1 e) 8 2 d) 2
c) 2 2 e) ∞

07. (PUC/SP) O limite da soma dos termos da


1 1 1  12. (UFC) Uma bola elástica cai de uma altura de
progressão geométrica  , , ,...  é:
 3 9 27  3 metros, elevando-se em cada pulo, a uma
1
1 2 altura igual a da que caiu anteriormente. Que
a) d) 5
2 3 espaço terá percorrido até ficar em repouso?
1 3 a) 1,5
b) e)
3 2 b) 2,5
1 c) 4,5
c)
4 d) 5,5
17 OS.:2196/13-Juliana
Prof. Fábio Frota

4 8 14
• Propriedade da função logarítmica
13. (Cescea/SP) Se 2   2  ...  , então o
m m 5 A função f é injetora, isto é: loga x = loga y  x = y
valor de m é: Para a > 1, a função é estritamente crescente isto é:
a) 5 loga x < loga y  x < y
b) 6 Para 0 < a < 1, a função é estritamente decrescente
c) 7 isto é: loga x < loga y  x > y
d) 8
e) 9
Exercícios de sala
EsPCEx – AFA – EFOMM – EEAR | MATEMÁTICA

Exponencial e logaritmos
01. Seja a função f : R  R definida por f(x)= 3x.
• Função exponencial Determine os valores de x R tais que
É toda função f : R  R+* definida por f(x) = ax, f(x + 1) + f(– x + 4) = 36.
com 0 < a  1.

02. Se x1 e x2 são raízes da equação


• Propriedade da Função exponencial
   
x x
A função f é injetora, isto é: ax = ay  x = y 3 2 3 2  2 3 calcule x21 + x22.
Para a > 1, a função é estritamente crescente isto é:
ax < ay  x < y
Para 0 < a < 1, a função é estritamente decrescente
isto é: ax < ay  x > y 03. Se x1 e x2 são as raízes da equação 3x + 3–x = 4,
sendo x1 > x2, calcule x1 – x2.
• Logaritmos
Denomina-se logaritmo do número b na base a,
04. Seja a função exponencial f(x) = 3 x . Se
e expoente x ao qual se deve elevar a para se obter b.
f (p)  f ( p) 1
a
b x
 loga b  x , em que a, b  R,  , determine p.
  f (p)  f ( p) 3
função exponencial função logarítimica
b>0e1a>0
05. Dados log 2 = 0,3 e log 3 = 0,4, resolva a equação
• Consequência da definição 9x – 7 . 3x + 10 = 0.
a) loga 1 = 0
b) loga a = 1
c) u loga b = b 6 x 1  6 x 2
06. Resolva a equação  1.
d) loga b = loga c  b = c 61 x  62 x

• Propriedades dos logaritmos


07. Resolva a equação
a) loga (b . c) = loga b + loga c
log2(9|x–2| + 7) = 2 +log2 (3|x–2| + 1).
b
b) loga   = loga b – loga c
c
c) loga bn = n . loga b 08. Seja D o mais amplo conjunto de números reais
1 x 1
d) log n b = Iog a b onde se pode definir f ( x )  log , x  D.
a n x5
Calcule o simétrico do maior número inteiro
e) cologa b = – loga b
logc b pertencente a D.
f) loga b = (mudança de base)
logc a
09. Se A = {x  R / log2 (x – 3) + log2 (x – 2) < 1} e
• Função logarítmica (a,b) é o menor intervalo que contém A, determine
É toda função f : R  R+* definida por f(x) = logax, o valor de a + b.
com a, x e R, 0 < a  1.

OS.:2196/13-Juliana
18
Prof. Fábio Frota

10. Se x 1 e x 2 são as soluções da equação 04. Considere a função real de variável real, definida
x 4 por f(x) = 3 + 2–x. Então f(log25) é igual a:
xlog5 x = , sendo x1 < x2, determine o valor de
125 4
1 a)
(x2 – x). 5
2 8
b)
5
12
c)
Exercícios de casa 5 5
16
d)
5

EsPCEx – AFA – EFOMM – EEAR | MATEMÁTICA


01. Se log7875 = a, então log35245 é igual a: e) 4

a+2
a) 05. Sejam logam = p e logan = q . Se p + q = logax e
a+7
a+2 p – q = logay, o valor de m2 é:
b)
a+5 a) xy
a+5
c) b) x2
a+2
a+7 c) y2
d)
a+2 d) x – y
x
a+5 e)
e) y
a+7

06. A opção em que figuram as soluções da equação


02. O número real que é raiz da equação
28   
dada por 3 x  log log  10 10 10 10    0 é:
5x + 2 + 5x – 1 + 5x + 1 + 5x = 780 é:   
a) 1 a) – 3 e 2
b) 2 b) – 3 e 3
c) 3 c) – 2 e 3
d) 4 d) – 2 e 2
e) 5 e) 2 e 3

03. Suponha que o nível sonoro  e a intensidade I 07. Se 2x = 8y+1 e 9x – 9, calcule o valor de x + y.
de um som estejam relacionados pela equação
logarítmica  = 120 + 10 log10I, em que  é medido
08. Determine o maior inteiro menor que a soma das
em decibéis e I, em watts por metro quadrado.
raízes da equação 4x – 90 . 2x + 89 = 0.
Sejam, I1 a intensidade correspondente ao nível
sonoro de 80 decibéis de um cruzamento de
duas avenidas movimentadas e, I2 a intensidade 09. Seja S o conjunto solução da inequação
correspondente ao nível de 60 decibéis do interior
log 1 (log3 x  log9 [ x  8])  0. Se Z representa
de um automóvel com ar-condicionado. A razão
3
I1 o conjunto dos números inteiros, então o conjunto
é igual a:
l2 Z  S possui exatamente:
1 a) 2 elementos
a)
10 b) 3 elementos
b) 1 c) 1 elemento
c) 10 d) 4 elementos
d) 100 e) 5 elementos
e) 1000

19 OS.:2196/13-Juliana
Prof. Fábio Frota

10. Se N é maior inteiro pertencente ao domínio da 15. A intensidade D de um terremoto, medida na escala
função f ( x )  1  log2 x , determine o valor de Richter, é um número dado pela fórmula empírica
N3 + 3N2 + 2. 2 E
D = . log na qual E é a energia liberada no
3 E0
terremoto, em kilowatt-hora, e E0 = 7 . 10–3 kWh.
11. O número real x, tal que x  23 log2 3  32log3 2 , é A energia liberada em um terremoto de intensidade
igual a: 4 na escala Richter, é em kilowatt-hora, um número
compreendido entre:
a) 36
a) 100.000 e 500.000
b) 42
b) 50.000 e 100.000
c) 54
EsPCEx – AFA – EFOMM – EEAR | MATEMÁTICA

c) 10.000 e 50.000
d) 60
d) 1.000 e 10.000
e) 72
e) 500 e 1.000

12. Seja f a função logarítmica dada por f(x) = logx, 16. Resolva a equação
para todo número real x > 0. É verdade que: 1 – logx – log(x – 1) + log(3x) = 0, considerando
a) o gráfico de f tem concavidade voltada para a base dos logaritmos igual a 10. A equação dada:
cima. a) não possui raiz real.
b) f(a + b) = f(a) + f(b) b) possui uma única raiz real, maior do que 30.
c) o gráfico da sua função inversa f–1 é simétrico c) possui duas raízes reais, diferentes, localizadas
em relação ao eixo das ordenadas. entre 10 e 20.
d) f–1 (a + b) = f–1 (a) . f–1(b), a e b reais. d) possui raízes reais, uma maior e outra menor
do que 1.
e) 0 < f(x) < 1 implica que 0 < x < 10.

17. Se x1 e x2 (x1 < x2), são as soluções da equação


13. Considere a função definida por y = logax, com
logx 2.log x 2 = log x 2 então logx1 x2 é igual a:
a > 0, a  1, e as sentenças seguintes:
64 64
I. y não assume valores negativos.
a) 4,5
II. Se 0 < a < 1, os valores de y decrescem à
medida que x cresce. b) 3,5
III. Se a > 1, o gráfico dessa função tem a c) 2,5
concavidade voltada para baixo. d) 1,5

Sobre essas sentenças, é correto concluir que: 18. Se x é um número real positivo e diferente de 1,
a) somente I e II são verdadeiras. 2
então o valor de xq, onde q = ,é:
log3 x
b) somente I e III são verdadeiras.
c) somente II e III são verdadeiras.
a) 5
d) I, II e III são verdadeiras.
b) 6
e) I, II e III são falsas.
c) 8
d) 9
14. No universo R, a equação 22+2x – 9 . 2x + 2 = 0
admite:
19. Se x1 e x2 são as raízes da equação
a) duas raízes inteiras negativas. 2
0, 7 x – 0,49x = 0, então |x1 – x2| é igual a:
b) quatro raízes inteiras.
c) uma única raiz. a) 1,5
d) duas raízes positivas, uma inteira e outra não b) 2
inteira. c) 3,5
e) duas raízes inteiras de sinais contrários.
d) 4

OS.:2196/13-Juliana
20
Prof. Fábio Frota

20. Se x2 – y2 = 80 e x – y = 8, então logxy(x + y) é a disjunção “ou”, pois ocorre quando temos que
aproximadamente igual a: satisfazer uma ou outra possibilidade. Por exemplo,
de um grupo de pessoas escolher um homem ou uma
a) 1
mulher.
b) 2
c) 3
Exemplo:
d) 4 Uma montadora de automóveis apresenta um
carro em três modelos diferentes (Hath, Sedan
e Perua), dois tipos de motores (1.0 e 1.6) e em
cinco cores diferentes (Azul, Branco, Cinza, Preto e
Análise combinatória – parte I Vermelho). Um consumidor terá quantas opções de

EsPCEx – AFA – EFOMM – EEAR | MATEMÁTICA


carros para escolher?
Mentalize
Introdução geral
Daremos início nessa aula a uma das partes
mais fascinantes da matemática, a chamada análise
combinatória. Ela surgiu da necessidade de calcular
o número de possibilidades existentes nos chamados
jogos de azar, depois foram percebendo o quão
importante era em outras situações. Essa parte da
Matemática estuda os métodos de contagem.
Esses estudos foram iniciados já no século XVI, Solução:
pelo matemático italiano Niccollo Fontana (1500- O número de opções é o produto das
1557), conhecido como Tartaglia. Depois vieram os possibilidades de cada evento, ou seja, MODELO x
franceses Pierre de Fermat (1601-1665) e Blaise MOTOR x COR.
Pascal (1623-1662).
A Análise Combinatória visa desenvolver 3 2 5 = 30 opções.
métodos que permitam contar (de uma forma indireta)
o número de elementos de um conjunto, estando Exemplo:
esses elementos agrupados sob certas condições. Uma moça se arrumava para sair com o
Veremos que a análise combinatória também funciona namorado, estando em dúvida se usava blusa, calça
como ferramenta indispensável para o estudo da e sapatos ou se iria de vestido e sapato. Se ela
estatística, da probabilidade, da genética, da física, provar todas as possíveis combinações de roupas,
dentre muitas outras aplicações. experimentando apenas as duas blusas, as duas
Princípio da contagem calças, os três pares de sapatos e os três vestidos
que ela está em dúvida, quantas combinações de
O princípio da contagem é sem dúvida o conceito
roupas ela terá provado?
mais importante da análise combinatória, pois é de
onde nasceram os fundamentos dessa disciplina.
A contagem pode ser dividida em dois princípios:
multiplicativo e aditivo.

 Princípio multiplicativo:
Você deve multiplicar o número de possibilidades
de cada evento obtendo o número de resultados
distintos do experimento composto. Esse
procedimento está associado a conjunção “e”, pois
ocorre quando temos que satisfazer possibilidades Solução:
simultaneamente. Por exemplo, de um grupo de Perceba que, ou ela vai de blusa, calça e sapato,
pessoas escolher um homem e uma mulher. ou ela vai de vestido e sapato. Então nós faremos
cada caso isolado e depois somamos, utilizando o
 Princípio aditivo: princípio aditivo.
Nesse caso devemos somar as possibilidades,
ou seja, calcular cada possibilidade isoladamente para 2 2 3 + 3 3
depois somar. Esse procedimento está associado blusa calça sapato vestido sapato

21 OS.:2196/13-Juliana
Prof. Fábio Frota

Portanto, Solução:
12 + 9 = 21 possibilidades Nesse caso as 3 letras vão ser embaralhadas.
Observa-se que existem 3 possibilidades (L, U e
Observação A) para a primeira posição, 2 possibilidades para
a segunda posição pois uma das letras já está na
Se a namorada gastar apenas 3 minutos em primeira posição e uma para a última, logo
cada combinação, passará mais de 1 hora apenas
escolhendo a roupa. Imaginem se somarmos o = 6
3 2 1
tempo do banho, da maquiagem, dos acessórios, anagramas
etc. O namorado tem que ter paciência!
Nesse caso, como são poucos resultados
EsPCEx – AFA – EFOMM – EEAR | MATEMÁTICA

Exemplo: também poderíamos até escrever cada um dos


A frota de veículos no Brasil é de pouco mais anagramas e contá-los.
de 30 milhões de veículos, mas nesse ritmo de LUA ALU ULA
crescimento da frota teremos 100 milhões em alguns LAU AUL UAL
anos. Quantas placas de carro diferentes nosso
sistema suporta?
Exemplo:
GOIÂNIA-GO (NCE) João recebeu o seguinte problema:
LEO – 0125 construa cartazes com quatro letras seguidas de três
números. As letras pertencem ao conjunto {I, B, G, E} e
podem ser usadas em qualquer ordem sem repetição.
Solução: Os números devem ser pares e pertencentes ao
As placas no Brasil são formadas por 3 letras conjunto {1, 2, 3, 4, 5, 6}, e também podem ser usados
(dentre 26 do alfabeto) e 4 números (algarismos de em qualquer ordem e sem repetição. O número de
0 a 9), logo cartazes diferentes que João pode confeccionar é:
a) 49
26 26 26 10 10 10 10 = 175.760.000 b) 72
c) 98
d) 120
Portanto, podemos ter mais de 175 milhões de
veículos na frota usando esse sistema. e) 144

Exemplo: Solução:
Quantas placas de carro no Brasil podem Os cartazes devem ter
começar com H e terminar com um número ímpar? __.__.__.__.__.__.__
L L L L P P P
FORTALEZA-CE
Observe os números são pares (não é um
HAB – 5227 número par de três algarismos e sim três números
pares), logo o produto das possibilidades será:
4 . 3 . 2 .1 . 3 . 2 . 1
Solução: Portanto
Temos 7 posições a serem ocupadas, a 144 possibilidades
primeira só uma possibilidade (H) e a última tem
5 possibilidades (1,3,5,7,9), a segunda e terceira
Exemplo:
posição terá 26 possibilidades cada (todo o alfabeto)
e as demais 10 possibilidades (algarismos de 0 a 9). (NCE) Uma “capícua” é um número que lido
de trás para diante é igual ao número original. Por
exemplo, 1881 é uma “capícua”, 134 não é “capícua”.
1 26 26 10 10 10 5 = 3.380.000
Usando apenas os algarismos 1, 2 e 3 , além de
11111, 22222 e 33333, há a seguinte quantidade de
Portanto, mais de 3 milhões de veículos. números de cinco algarismos que são “capícuas”:
a) 6; d) 20;
Exemplo: b) 12; e) 24.
Existem quantos anagramas da palavra LUA? c) 16;

OS.:2196/13-Juliana
22
Prof. Fábio Frota

Solução: 03. Determine quantos números de 3 algarismos


Vamos calcular o total de “capícuas” pelo podemos formar utilizando apenas os algarismos
princípio da contagem. 2, 3, 4, 5, 6, 7 e 9, de forma que figurem pelo
menos dois algarismos iguais.
a) 343 c) 133
b) 210 d) 90

Então pelo produto das possibilidades, temos: 04. Quantos números pares de três algarismos
3 . 3 . 3 . 1 . 1 = 27 possibilidades distintos podemos formar utilizando apenas os
algarismos 2, 3, 4, 5, 6, 7 e 9?

EsPCEx – AFA – EFOMM – EEAR | MATEMÁTICA


Excluindo-se os números 11111, 22222 e 33333, a) 343 c) 133
temos:
b) 210 d) 90
24 possibilidades

Palíndromo ou capícua 05. Quantos números de 3 algarismos distintos são


Quando um texto, ou número, é denominado de maiores que 500, utilizando apenas os algarismos
Capícua (ou palíndromo), significa que ele pode ser 2, 3, 4, 5, 6, 7 e 9?
lido de duas maneiras simétricas, ou seja, do início a) 240 c) 120
para o fim ou de trás pra diante.
b) 210 d) 90

Olhem que texto interessante!


06. Determine quantos números pares de três
Este texto de Clarice Lispector tem sentido duplo,
algarismos distintos são maiores que 500,
um quando lido de cima para baixo e um sentido
utilizando apenas os algarismos 2, 3, 4, 5, 6, 7
exatamente o contrário quando lido de baixo para
e 9.
cima.
“Não te amo mais. a) 65 c) 45
Estarei mentindo dizendo que b) 55 d) 35
Ainda te quero como sempre quis.
Tenho certeza que
Nada foi em vão.
Sinto dentro de mim que Exercícios de casa 6
Você não significa nada.
Não poderia dizer jamais que 01. Quantos números distintos, de quatro algarismos,
Alimento um grande amor. podemos formar, utilizando apenas os algarismos
Sinto cada vez mais que 0, 1, 2, 3, 4, 5 e 6, de forma que ele seja ímpar e
Já te esqueci! menor que 4000?
E jamais usarei a frase
a) 114 c) 441
EU TE AMO!
Sinto, mas tenho que dizer a verdade b) 140 d) 882
É tarde demais...”
02. Utilizando apenas os algarismos 0, 1, 2, 3, 4, 5 e
Exercícios de sala 6, podemos formar quantos números ímpares de
quatro algarismos distintos que sejam menores
que 4000?
01. Quantos números de 3 algarismos podemos
formar utilizando apenas os algarismos 2, 3, 4, 5, a) 90 c) 140
6, 7 e 9? b) 120 d) 210
a) 343 c) 133
b) 210 d) 90 03. Determine quantos números pares de três
algarismos são maiores que 600, utilizando
02. Determine quantos números de três algarismos apenas os algarismos 0, 3, 4, 6, 7 e 9.
distintos podemos formar utilizando apenas os a) 63
algarismos 2, 3, 4, 5, 6, 7 e 9. b) 53
a) 343 c) 133 c) 43
b) 210 d) 90 d) 33
23 OS.:2196/13-Juliana
Prof. Fábio Frota

04. Determine quantos números pares de três 07. Na figura abaixo temos um esboço de parte do
algarismos distintos são menores que 600, centro da cidade do Recife com suas pontes.
utilizando apenas os algarismos 0, 3, 4, 6, 7 e 9. As setas indicam o sentido permitido do fluxo
a) 36 de tráfego de veículos. De quantas maneiras,
utilizando apenas o esboço, poderá uma pessoa
b) 30
ir de carro do ponto A ao ponto B e retornar ao
c) 26 ponto de partida passando exatamente por três
d) 20 pontes distintas?

05. Uma placa de automóvel é formada por três


letras e quatro números. Quantas placas podem
EsPCEx – AFA – EFOMM – EEAR | MATEMÁTICA

ser formadas apenas com vogais e apenas com


algarismos ímpares, como por exemplo EAA1355?
a) 75
b) 25
c) 55
d) 57

06. Determine a quantidade de placas de carro que


possuem três vogais, seguidas de quatro números. a) 8
a) 1.250.000 c) 320.000 b) 13
b) 630.000 d) 84.000 c) 17
d) 18
Recife, a Veneza brasileira e) 20

Recife é um município brasileiro, capital do 08. Quantos números naturais de seis algarismos
Estado de Pernambuco. Localizada às margens distintos podem ser formados com 1, 2, 3, 4, 5 e
do oceano Atlântico, a cidade tem uma área de 7 de modo que os algarismos pares nunca fiquem
217,494 km² e uma população de 1.561.659 juntos?
de pessoas (ou 3,73 milhões, contando a área
a) 720
metropolitana). Em recente estudo do IBGE, o
Recife aparece como metrópole da quarta maior b) 480
rede urbana do Brasil em população. O Recife c) 240
é conhecido como “Veneza Brasileira” graças d) 120
à semelhança fluvial com a cidade europeia de
Veneza. Cercado por rios e cortado por pontes,
é cheio de ilhas e mangues. Ali acontece o 09. Utilizando apenas os algarismos 1, 2, 3, 4, 5 e 6,
encontro dos rios Beberibe e Capibaribe que determine quantos números de quatro algarismos
deságuam no Oceano Atlântico. O município podem ser formados, de forma que os algarismos
conta com dezenas de pontes, entre elas a mais vizinhos nunca se repitam.
antiga do Brasil, a ponte Maurício de Nassau. a) 750
SITE WIKIPEDIA - http://pt.wikipedia.org/wiki/Recife
b) 720
c) 360
d) 120

10. Quantos números ímpares de cinco algarismos,


menores que 66400, podem ser formados a partir
dos dígitos 2, 3, 6, 7 e 9?
a) 930
b) 915
c) 900
Uma das inúmeras pontes da cidade do Recife
d) 750
OS.:2196/13-Juliana
24
Prof. Fábio Frota

Análise combinatória - parte II Permutação simples


Quando temos n objetos para ocupar n lugares
Mentalize devemos permutá-los de lugar e obteremos Pn
possibilidades distintas, ou seja

FATORIAL
Pn = n! para n ∈ N*
Introdução
O produto fatorial vai nos auxiliar na solução
de problemas de uma forma abreviada, será muito Exemplo:
importante para compreensão de outros conteúdos Foram escolhidas as 5 melhores redações em

EsPCEx – AFA – EFOMM – EEAR | MATEMÁTICA


também. um concurso promovido pelo jornal Diário e será feita
uma foto com os seus 5 autores, para divulgação do
Definição evento. De quantas maneiras distintas o fotografo do
Seja n um número natural, com n ≥ 2, indicamos Diário pode organizar esses 5 alunos para fotografá-
por n! como o produto de n pelos números naturais los?
positivos menores que n, isto é:

n! = n.(n – 1).(n – 2)...1

Exemplo:
2! = 2.1 = 2
3! = 3.2.1 = 6 Solução:
4! = 4.3.2.1 = 24 Como são cinco pessoas para cinco lugares,
5! = 5.4.3.2.1 = 120 caracterizamos a permutação (troca, no sentido de
embaralhar).
Observação 1: Temos então P5 = 5! = 5.4.3.2.1 = 120

Por convenção 1! = 1 e 0! = 1. Anagrama


Um anagrama (do grego ana = “voltar” ou “repetir”
Observação 2: + graphein = “escrever”) é uma espécie de jogo de
palavras, resultando do rearranjo das letras de uma
palavra ou frase para produzir outras palavras,
A propriedade fundamental dos fatoriais diz
utilizando todas as letras originais exatamente uma
que
vez.
n! = n.(n − 1)! , para n ∈ N , n ≥ 3
Um exemplo conhecido é o nome da personagem
IRACEMA, claro anagrama de AMÉRICA, no romance
Exemplo: de José de Alencar.
10! = 10.9! Em análise combinatória, os anagramas
de uma palavra não necessariamente precisam ter
15! = 15.14.13!
sentido gramatical, mas sim uma recombinação
20! = 20.19.18.17! aleatória das letras dessa palavra, usando cada uma
delas uma única vez.
Exemplo:
Simplifique os fatoriais: Exemplo:
Existem quantos anagramas da palavra LUA?
10 ! 10 . 9 . 8 !
a)   10 . 9  90
8! 8! Solução:
7! . 9! 7 . 6 . 5! . 9 . 8! Resolvemos esse problema anteriormente
b)   7 . 6 . 9  378
8! . 5! 8! . 5! usando princípio da contagem, no entanto podemos
n! n . (n  1) . (n  2)! simplesmente que o número de possibilidades é a
c)   n . (n  1)  n2  n permutação de 3 letras, ou seja, P3 = 3! = 3.2.1 = 6.
(n  2)! (n  2)!
(n  1)! (n  1)! 1 1
d)    Exemplo:
(n  1)! (n  1) . n . (n  1)! (n  1) . n n2  n
Quantos anagramas possui a palavra BOLA?

25 OS.:2196/13-Juliana
Prof. Fábio Frota

Solução: 04. Determine quantos anagramas da palavra CHUVA


Observe todos os anagramas da palavra bola. não possuem as vogais juntas.
a) 120
b) 72
c) 48
d) 24

05. Quantos anagramas da palavra CHUVA possuem


as consoantes juntas e em ordem alfabética?
Veja que dentre eles está a palavra LOBA, mas
EsPCEx – AFA – EFOMM – EEAR | MATEMÁTICA

não é preciso ter sentido gramatical, o importante é a) 12


encontrar o total de anagramas permutando essas 4 b) 10
letras, ou seja, c) 8
P4 = 4! = 4.3.2.1 = 24.
d) 6

Curiosidade Exercícios de casa 7


De aorcdo com uma pqsieusa de uma
01. Quantos anagramas possui a palavra LÓGICA?
uinrvesriddae ignlsea, não impotra em qaul odrem
as lrteas de uma plravaa etãso, a úncia csioa a) 120
iprotmatne é que a piremria e útmlia lrteas etejasm b) 480
no lgaur crteo. O rseto pdoe ser uma ttaol bçguana
c) 560
que vcoê pdoe anida ler sem pobrlmea. Itso é
poqrue nós não lmeos cdaa lrtea isladoa, mas a d) 720
plravaa cmoo um tdoo.
02. Considere todos os anagramas da palavra
FORTALEZA que começam e terminam pela letra
A. Quantos deles têm as consoantes juntas?
Exercícios de sala a) 5040
b) 1440
01. Quantos são os anagramas da palavra CHUVA? c) 720
a) 120 d) 480
b) 100
c) 80 03. Quantos anagramas da palavra JANEIRO tem as
d) 60 consoantes juntas e em ordem alfabética?
a) 24
02. Determine a quantidade de anagramas da palavra b) 52
CHUVA que começam e terminam por vogal. c) 120
a) 10 d) 720
b) 12
c) 14 04. De quantas maneiras Amanda, Bruno, Caio,
d) 16 Débora, Érica e Felipe, podem se organizar lado
a lado para tirar uma foto, sabendo que Caio
03. Quantos anagramas da palavra CHUVA possuem e Débora namoram e ficarão necessariamente
as vogais juntas? juntos?

a) 96 a) 120
b) 64 b) 240
c) 48 c) 360
d) 24 d) 720

OS.:2196/13-Juliana
26
Prof. Fábio Frota

05. Sete modelos, entre elas Ana, Beatriz, Carla e 08. (ENEM) Os alunos de uma escola organizaram um
Denise, vão participar de um desfile de modas. A torneio individual de pingue-pongue nos horários
promotora do desfile determinou que as modelos dos recreios, disputado por 16 participantes,
não desfilarão sozinhas, mas sempre em filas segundo o esquema abaixo:
formadas por exatamente quatro das modelos.
Além disso, a última de cada fila só poderá ser ou
Ana, ou Beatriz, ou Carla ou Denise. Finalmente,
Denise não poderá ser a primeira da fila. Assim,
o número de diferentes filas que podem ser
formadas é igual a:

EsPCEx – AFA – EFOMM – EEAR | MATEMÁTICA


a) 420
b) 480
c) 360 Foram estabelecidas as seguintes regras:
d) 240
• Em todos os jogos, o perdedor será eliminado;
06. Três rapazes e duas moças vão ao cinema e • Ninguém poderá jogar duas vezes no mesmo
dia;
desejam sentar-se, os cinco, lado a lado, na
• Como há cinco mesas, serão realizados, no
mesma fila. O número de maneiras pelas quais
máximo, 5 jogos por dia.
eles podem distribuir-se nos assentos de modo
que as duas moças fiquem juntas, uma ao lado da
Com base nesses dados, é correto afirmar que o
outra, é igual a?
número mínimo de dias necessário para se chegar
a) 2 ao campeão do torneio é:
b) 4 a) 8
c) 24 b) 7
d) 48 c) 6
d) 5
07. Para responder a certo questionário, preenche-se e) 4
o cartão apresentado a seguir, colocando-se um
“x” em uma só resposta para cada questão. De 09. Um “Pangrama” é uma frase que usa todas as
quantas maneiras distintas pode-se responder a letras do alfabeto, ou seja, uma combinação de
todas as letras do nosso alfabeto oficial, podendo
esse questionário?
ou não repeti-las. Admitem-se frases que não
possuem K, Y ou W, pois foram recentemente
introduzidas pela reforma ortográfica.

EXEMPLOS DE PANGRAMA

Em Português

“Um pequeno jabuti xereta viu dez cegonhas felizes.”

“Gazeta publica hoje no jornal uma breve nota de


faxina na quermesse.”

“Blitz prende Wyskie, vesgo com cheque fajuto.”


a) 3125
“Vi o ímã cai no pé do pinguim queixoso e Kowalsky
b) 120 pôr açúcar no chá do jabuti feliz.”
c) 32
d) 25 Em Inglês

e) 10 “The quick brown fox jumps over the lazy dog.”


27 OS.:2196/13-Juliana
Prof. Fábio Frota

Em Espanhol Sudoku – Origem: Wikipédia,


a enciclopédia livre.
“David exige plazo fijo embarque truchas y niños
New York” Objetivo
O objetivo do jogo é completar todos os quadrados
Quantas combinações de 26 letras são possíveis utilizando números de 1 a 9. Para completá-los,
de formar, sem repetir as letras, formando ou não
seguiremos a seguinte regra: Não podem haver
uma frase com sentido e desprezando os espaços
números repetidos nas linhas horizontais e verticais,
entre letras?
assim como nos quadrados grandes.
a) 2610
b) 1026
EsPCEx – AFA – EFOMM – EEAR | MATEMÁTICA

c) 226
d) 26!

10.
O QUE É SUDOKU?

Como jogar
O jogo é mais frequentemente uma grade de
Sudoku, por vezes escrito Su Doku, é um
9×9 constituída de sub-grades de 3×3 chamadas
puzzle baseado na colocação lógica de números.
de regiões (outros termos incluem caixas, blocos,
O objetivo do puzzle é a colocação de números de
algumas vezes porém o termo quadrante é utilizado,
1 a 9 em cada uma das células vazias numa grelha
apesar de ser um termo impreciso para uma
de 9×9, constituída por 3×3 subgrelhas chamadas
grade de 3×3). Alguma célula já contém números,
regiões. O puzzle contém algumas pistas iniciais.
chamadas como números dados (ou algumas vezes
Cada coluna, linha e região só pode ter um número
pistas). O objetivo é preencher as células vazias,
de cada um dos 1 a 9. Resolver o problema
com um número em cada célula, de maneira que
requer apenas raciocínio lógico e algum tempo.
cada coluna, linha e região contenham os números
Os problemas são normalmente classificados em
1–9 apenas uma vez. Portanto, na solução do jogo,
relação à sua realização. O aspecto do puzzle
cada número aparece apenas uma vez em qualquer
Sudoku lembra outros puzzles de jornal.
um dos sentido ou regiões, daí portanto “únicos
SITE WIKIPEDIA - http://pt.wikipedia.org/wiki/sudoku números” originaram o nome do jogo ou enigma.

Determine quantas maneiras distintas podemos


Métodos de solução
preencher uma célula do Sudoku, de acordo com
as regras descritas. A região 3×3 no canto superior direito. O
solucionador pode eliminar todas as células vazias
a) 109
no canto superior direito que contenham um 5 nas
b) 9!
mesmas colunas ou linhas. Isto deixa apenas uma
c) 910
célula possível (destacada em verde).
d) 29

OS.:2196/13-Juliana
28
Prof. Fábio Frota

difíceis, por definição, não podem ser resolvido pela


varredura básica somente.

Marcações
Fazer a varredura termina quando mais nenhum
número adicional pode ser descoberto. Deste ponto
em diante, é necessário fazer algumas análise
lógicas. Muitos acham útil guiar esta análise através
da marcação dos números possíveis (candidatos) nas
células em branco. A forma mais popular é notação
subscrita.

EsPCEx – AFA – EFOMM – EEAR | MATEMÁTICA


A estratégia para resolver um enigma pode ser
considerada como compreender uma combinação
de três processos: fazer uma varredura visual, fazer
marcações, e análise.

Varredura Na notação subscrita, os números possíveis


A varredura é executada no início e durante são escritos a lápis em tamanho reduzido
toda a solução. As varreduras somente têm que ser (subscritos) nos cantos das células em branco.
executadas uma vez entre períodos da análise. A O inconveniente a este é que os puzzles
varredura consiste em apenas duas técnicas básicas: originais impressos em um jornal são geralmente
• Cruzamento: a varredura das linhas (ou demasiado pequenos para acomodar mais do que
colunas) para identificar que linha em uma região
alguns dígitos da escrita normal. Mas na prática,
particular pode conter um determinado número
não precisamos usar mais de quatro dígitos
por um processo do eliminação. Este processo
subscritos.
é repetido então com as colunas (ou linhas).
Para resultados mais rápidos, os números Fácil
são verificados por ordem de frequência. É
importante executar sistematicamente este
processo, verificando todos os dígitos 1–9.
Contar de 1–9 nas regiões, linhas, e colunas
para identificar os números faltantes. Contar
baseada no último número descoberto pode fazer
com que a busca seja mais rápida. Também pode ser
o caso (tipicamente em enigmas mais difíceis) que a
maneira a mais fácil verificar o valor de uma célula
individual é contando no inverso — isto é, fazendo
a varredura da região da célula, linha, e coluna para
identificar os valores que não podem ser, a fim de se
descobrir o que resta.
Os solucionadores avançados procuram
contingências ao fazer a varredura —isto é, estreitando
a posição de um numeral dentro de uma fileira,
coluna, ou região a duas ou três células. Quando
estas células todas se encontrarem dentro da mesma
fileira (ou coluna) e região, elas podem ser usadas
para finalidades de eliminação durante as etapas de
cruzamento e contar. Particularmente os enigmas mais
desafiadores podem requerer múltiplas contingências
para serem descobertos, talvez em direções múltiplas
ou mesmo cruzamentos múltiplos. Os enigmas que
podem ser resolvidos apenas fazendo-se a varredura
sem necessidade de detectar as contingências são
classificados como enigmas fáceis; enigmas mais
29 OS.:2196/13-Juliana
Prof. Fábio Frota

Médio Exemplo:
Lançando-se uma moeda seis vezes, quantas
sequências diferentes de resultados apresentam
quatro caras e duas coroas?

Solução:
Existem casos em que você pode usar a
permutação com repetição para resolver a questão,
criando um anagrama para representar a situação.
Chamando cara e coroa de letras diferentes, como
K e C, respectivamente, podemos escrever uma das
EsPCEx – AFA – EFOMM – EEAR | MATEMÁTICA

sequências como KKKKCC.


O número de total sequências é igual ao no de
anagramas de KKKKCC, ou seja,
9!
P94,5 = = 15 sequências diferentes de
4 ! . 2!
resultados.

Exemplo:
A figura abaixo mostra um mapa de uma pequena
parte da cidade de Fortaleza. Quando Ribamar vai
de casa (esquina 1) até o shopping Aldeota (esquina
2), ele percorre exatos 9 quarteirões. Na figura, está
representada apenas uma das várias possibilidades
de caminhos que ele pode escolher. Determine
quantos caminhos diferentes, sem voltar, ele pode
escolher para ir de casa até o shopping.
Solução (digite o código):

www.sudoku-puzzles.net

Análise combinatória – parte III

Mentalize
Permutação com repetição
Quando existem elementos repetidos a serem
permutados, as trocas de lugares entre eles, não
influenciam, portanto não podem ser contadas. Temos
então:
n!
Pnn1,n2 ,...,nk =
n1 !. n2 !...nk !

Solução:
Onde n é o total de elementos a ser permutado e
nk é o número de vezes que cada elemento se repetiu. Observe que ele anda 5 vezes para oeste (O) e
4 vezes para o sul (S), veja que na figura a sequência
Exemplo:
Existem quantos anagramas da palavra SOOSOOSSO
BANANA?
Solução: Portanto, o número de caminhos possíveis é
Observe que a letra A aparece 3 vez, enquanto o igual ao número de anagramas da sequência
N aparece 2 vezes e o B só 1 vez. Portanto
6! SSSSOOOOO
P61, 3, 2 = = 60 anagramas
1! . 3 ! . 2!

OS.:2196/13-Juliana
30
Prof. Fábio Frota

Ou seja
Exercícios de sala
9!
P94,5 =
4 ! . 5 ! = 126 caminhos diferentes.
01.
Exemplo: Cantigas de roda
Quantos números pares obtemos permutando-se
os algarismos 1, 2, 2, 3, 3, 3 e 4?

Solução:
Para que o número seja par, deve terminar em

EsPCEx – AFA – EFOMM – EEAR | MATEMÁTICA


número par, ou seja, dentre esses, apenas 2 ou 4.

Terminando em 2, temos a permutação dos


outros algarismos 123334, logo

3 6 ! 6 .5 .4 .3 !
P=
6 = = 120
3! 3!

Terminando em 4, temos a permutação dos outros


As cantigas de rodas, cirandas ou
algarismos 122333, logo
brincadeiras de roda são brincadeiras infantis,
3 6 ! 6 .5 .4 .3 ! onde tipicamente as crianças formam uma roda
P=
6 = = 120 de mãos dadas e cantam melodias folclóricas,
3! 3!
podendo executar ou não coreografias acerca da
Portanto, somando as possibilidades temos um letra da música. É uma grande expressão folclórica
total de 180 números. e, acredita-se que pode ter origem em músicas
modificadas de um autor popular ou nascido
Permutação circular anonimamente na população. Entre as cantigas
de roda mais conhecidas estão “Roda pião”,
Nesse caso é só fixar um dos elementos em seu
“Escravos de Jó”, “Rosa juvenil”, “Sapo Cururu”,
lugar para ter um referencial.
“O cravo e a rosa” e “Atirei o pau no gato”. Como
o intuito é a brincadeira, carregam uma melodia
P(n–1) = (n–1)! de ritmo limpo e rápido, favorecendo a imediata
assimilação.
Exemplo:
De quantas maneiras distintas 6 pessoas podem SITE WIKIPEDIA - http://pt.wikipedia.org/wiki/

sentar-se em uma mesa redonda? Cantigas_de_roda

Solução:
Se um grupo de oito crianças deseja brincar de
Imagine se todos mudassem para cadeira ao seu
ciranda, quantas configurações diferentes podem
lado! Você não teria nenhuma mudança, afinal todos
ser formadas para essa roda?
continuariam vizinhos as mesmas pessoas. Então,
nesse caso fixa-se uma das pessoas e permuta-se as a) 120
outras 5, logo b) 720
P5 = 5! = 120 possibilidades. c) 5040
d) 40320
Exemplo:
De quantas maneiras distintas 4 pessoas podem
sentar-se em uma mesa quadrada? 02. De quantas maneiras distintas seis pessoas
podem sentar-se ao redor de uma mesa redonda,
Solução:
de modo que A e B fiquem lado a lado?
Para ocorrer uma permutação circular, não é
preciso que seja em uma mesa redonda. a) 6
Na mesa quadrada, também devemos fixar um b) 24
dos quatro e permutar os outros três, logo c) 48
P3 = 3! = 3.2.1 = 6 d) 72

31 OS.:2196/13-Juliana
Prof. Fábio Frota

03. Existem quantos anagramas da palavra


SUCESSO, que começam com C e terminam
com O?
a) 12
b) 20
c) 24
d) 60

04.
a) 1
A bandeira
EsPCEx – AFA – EFOMM – EEAR | MATEMÁTICA

b) 2
c) 3
d) 4

03. De quantas maneiras podemos organizar lado a


lado, 3 garrafas idênticas e 2 copos idênticos?
a) 120
A bandeira é definida classicamente b) 24
como sendo o símbolo representativo de um c) 10
estado soberano, ou país, município, intendência, d) 6
província, organização, sociedade, clã, coroa,
e) 5
reino, ou seja, todo ente constituído seja uma
nação e seu povo, até mesmo uma família
tradicional, desde que reconhecidos pelos entes, 04. Um salão possui 4 portas, onde cada uma delas
por Lei ou tradição. A bandeira acima, está pode está aberta ou fechada. De quantas maneiras
dividida em seis faixas que serão pintadas de azul, podemos deixar esse salão aberto?
vermelho e branco. a) 64
SITE WIKIPEDIA – http://pt.wikipedia.org/wiki/Bandeira b) 32
Determine quantas bandeiras distintas poderão c) 31
ser criadas, sabendo que exatamente três faixas d) 16
devem ser azuis, duas vermelhas e uma branca. e) 15
a) 60
b) 90 05. O curador de um museu dispõe de três quadros de
c) 120 Michelangelo e três de Portinari e quer expô-los
em uma mesma parede, lado a lado. Todos os seis
d) 150
quadros são assinados e datados. Para o curador,
os quadros podem ser dispostos em qualquer
Exercícios de casa 8 ordem, desde que os de Michelangelo apareçam
ordenados entre si em ordem cronológica, da
01. De quantas maneiras distintas seis pessoas esquerda para a direita. O número de diferentes
podem sentar-se em uma mesa redonda? maneiras que os seis quadros podem ser expostos
é igual a:
a) 6
b) 24
c) 120
d) 720

02. Quatro amiguinhos, Anna, Pedro, Carine e


Daniel brincam de roda no parque, como mostra
“A criação de Adão” de Michelangelo
a figura abaixo. De quantas maneiras distintas
eles podem se organizar para brincar nessa a) 20 d) 120
roda, de forma que as crianças de mesmo sexo b) 30 e) 360
não fiquem lado a lado? c) 24
OS.:2196/13-Juliana
32
Prof. Fábio Frota

06. Lançando-se uma moeda seis vezes, quantas A população da China é a maior do mundo,
sequências diferentes de resultados apresentam somando mais de 1 350 milhões (ou 1,35 bilhão)
quatro caras e duas coroas? de habitantes, distribuídos entre a República
Popular da China, com mais de 1 330 milhões
a) 64
(ou 1,33 bilhão) de pessoas, e Taiwan, com mais
b) 32
de 20 milhões de habitantes. Trata-se da maior
c) 20 população do planeta e representa mais de um
d) 15 quinto do total mundial.
e) 10 SITE WIKIPEDIA - http://pt.wikipedia.org/wiki/China

07. Uma sala possui três portas, de quantas maneiras

EsPCEx – AFA – EFOMM – EEAR | MATEMÁTICA


Sichuan é o nome de um território chinês e da
diferentes podemos deixar a sala aberta? sua principal cidade. Determine em quantos
a) 8 anagramas da palavra SICHUAN aparece a
b) 7 palavra CHINA em alguma posição?
c) 6 a) 1
d) 5 b) 6
e) 4 c) 24
d) 120
08.
China, maior população do mundo 09. Determine o número de anagramas da palavra
ALAMEDA que não apresenta as 4 vogais juntas.
a) 96
b) 744
c) 816
d) 840
e) 960

10. Uma das prateleiras da livraria “Intelecto”, possui


] cinco livros iguais de Raciocínio Lógico, quatro
livros iguais de Informática e três livros diferentes
de Direito. De quantas maneiras podem ser
organizados esses doze livros, se eles ficarem
China ( em chinês tradicional, em chinês dispostos lado a lado e em qualquer ordem.
simplificado, Zhongguó no sistema pinyin e
a) 364800
Chung-kuo no sistema Wade-Giles) é uma região
cultural, uma antiga civilização e, dependendo b) 281600
da perspectiva, uma entidade nacional ou c) 166320
multinacional, situada na maior parte do leste d) 83400
asiático. Da última Guerra Civil Chinesa (cujo e) 14120
maior combate terminou em 1949) resultou na
formação de duas entidades políticas que usam Anotações
o nome China:
• República Popular da China (RPC),
vulgarmente chamada de China Comunista
ou simplesmente China, tem controle sobre
a China continental e sobre os territórios
auto-governados de Hong Kong (desde
1997) e Macau (desde 1999).
• República da China (RDC), vulgarmente
chamada de China Nacionalista ou Taiwan,
tem controle sobre as ilhas de Taiwan,
Pescadores, Kinmen e Matsu.

33 OS.:2196/13-Juliana
Prof. Fábio Frota

Análise combinatória – parte IV n!


Cn,p  para {n,p} ⊂ N*, com n >p
p !.(n  p)!
Mentalize
Arranjo simples Observação
Se temos n elementos para ocupar p posições
que ainda podem permutar, ou seja, importando a Como no Arranjo importa a ordem dos
ordem, temos An,p possibilidades distintas (lê-se elementos e na Combinação não importa, teremos
arranjo de “n” elementos tomados “p” a “p”). sempre um número de arranjos maior ou igual ao
EsPCEx – AFA – EFOMM – EEAR | MATEMÁTICA

de combinações. An,p  Cn,p.


n!
A n,p  para {n,p} ⊂ N*, com n>p
(n  p)!
Exemplo:
Exemplo: Em um desfile de moda com 8 semifinalistas,
Em um desfile de moda com 8 finalistas, o júri o júri deve escolher 3 para serem as finalistas que
deve escolher 3 para serem eleitas como rainha, concorrem ao título de rainha. De quantas maneiras
princesa e miss simpatia. De quantas maneiras diferentes podemos ter esse resultado?
diferentes podemos ter esse resultado?
Solução:
Solução: Temos aqui uma combinação de 8 pessoas
Temos aqui um arranjo de 8 pessoas tomadas 3 tomadas 3 a 3, pois será um conjunto de 3 pessoas,
a 3, pois importa a ordem, logo logo não importa a ordem, portanto

8! 8 ! 8 .7 .6 .5 ! 8! 8! 8 . 7 . 6 . 5!
C8,3     8 . 7  56
A 8,3     8.7.6  336 3 ! . (8  3)! 3 ! . 5 ! 3 . 2 . 1 . 5 !
(8  3)! 5 ! 5!

Teremos 336 possibilidades distintas Teremos 56 possibilidades distintas

Exemplo:
Exemplo:
Quatro amigos vão ao cinema e escolhem, para
A escrita Braille para cegos é um sistema de
sentar-se, uma fila em que há seis lugares disponíveis.
símbolos onde cada caractere é formado por uma
Sendo n o número de maneiras como poderão sentar-
se, o valor de n/5 é igual a: matriz de 6 pontos dos quais pelo menos um se
destaca em relação aos outros. Assim por exemplo:
Solução:
A C
Observe que essa questão é resolvida da mesma
forma que 6 pessoas para 4 cadeiras, logo, como
importa a ordem, temos um arranjo de 6 posições
tomadas 4 a 4, portanto
6! 6 ! 6 .5 .4 .3 .2 ! Qual o número máximo de caracteres distintos
n  A 6,4     6.5.4.3  360
(6  4)! 2! 2! que podem ser representados neste sistema de
Teremos n=360 e dessa forma n/5 = 72 escrita?

Análise combinatória – parte V Solução::


Vamos resolver essa questão de duas formas:
usando combinação e usando princípio da contagem.
Combinação • Por combinação:
Se temos n elementos para formar um conjunto
A questão é saber quantos conjuntos de 1, 2,
com p posições, ou seja, sem importando a ordem
3, 4, 5 ou 6 elementos podemos formar com
dos elementos escolhidos, temos Cn,p possibilidades
esses seis pontos, ou seja, somar todas as
distintas (lê-se combinação de “n” elementos tomados
combinações possíveis.
“p” a “p”).

OS.:2196/13-Juliana
34
Prof. Fábio Frota

C6,1 + C6,2 + C6,3 + C6,4 + C6,5 + C6,6 02. Uma seleção possui 8 candidatos
para 3 vagas de vendedor de
6! 6! 6! 6!
    uma loja. De quantas maneiras
1!(6  1)! 2!(6  2)! 3 !(6  3)! 4 !(6  4)!
pode ser feita essa escolha?
6! 6!
  a) 24
5 !(6  5)! 6 !(6  6)!
b) 56
6 + 15 + 20 + 15 + 6 + 1 = 63 c) 336
d) 1444
Portanto, através dessa escrita é possível
representar 63 letras, números e símbolos diferentes. 03. De um grupo de 8 engenheiros e 6 arquitetos,

EsPCEx – AFA – EFOMM – EEAR | MATEMÁTICA


• Por princípio da contagem: serão escolhidos três funcionários para representar
Dessa forma fica bem mais fácil! É só imaginar a construtora Alfa em uma reunião, sendo 3
que existem 6 pontos, onde cada um deles engenheiros ou 3 arquitetos. Quantos grupos
só tem 2 opções (destacado ou não), então diferentes poderão ser formados?
fazendo o produto das possibilidades, temos:
a) 20 c) 76
2 . 2 . 2 . 2 . 2 . 2 = 64
b) 56 d) 1120

Esse resultado inclui todas as possibilidades,


então devemos excluir quando todos os pontos 04. A construtora Alfa possui 8 engenheiros e
não estiverem destaca, portanto existem 6 arquitetos, dos quais serão escolhidos 3
engenheiros e 3 arquitetos para projetar o
64 – 1 = 63 símbolos diferentes
empreendimento Beta. Quantas equipes diferentes
poderão ser formadas para esse empreendimento?
Visão além do alcance
a) 20 c) 76
b) 56 d) 1120
Olhe fixamente por mais de 30 segundos para a
figura a seguir e depois olhe para uma parede branca.
O que você está vendo agora? Jesus? 05. Uma construtora possui 8 engenheiros e 6
arquitetos. Quantas equipes, com três profissionais,
poderão ser formadas, de forma que figure nessa
equipe pelo menos um engenheiro e pelo menos
um arquiteto?
a) 560 c) 364
b) 480 d) 288

06. Uma construtora deverá distribuir 8 engenheiros


em três equipes: A, B e C. De quantas maneiras
poderá ser feita essa divisão, de modo que A e
B tenham três profissionais e a equipe C tenha
somente dois?
a) 560
b) 480
c) 364
Faça agora d) 288

01. De um grupo de 8 candidatos serão escolhidos 3


para ser o gerente, o caixa e o vendedor de uma Pratique mais 9
loja. De quantas maneiras pode ser feita essa
01. Um grupo de 10 empresários cumprimenta-se
escolha?
com apertos de mãos no início de uma reunião.
a) 24 Sabendo que cada um deles cumprimentou a
b) 56 todos, determine o número de apertos de mão.
c) 336 a) 90 c) 50
d) 1444 b) 70 d) 45
35 OS.:2196/13-Juliana
Prof. Fábio Frota

02. Abaixo temos um grupo de dez pessoas. 06. Caso 5 servidores em atividade e 3 aposentados
Determine de quantas maneiras distintas posso se ofereçam como voluntários para a realização
escolher 4 delas para ganhar uma passagem de de um projeto que requeira a constituição de
ida e volta, Fortaleza-Pacatuba, para visitar o uma comissão formada por 5 dessas pessoas,
Parque das Andréas, sabendo que a mãe que das quais 3 sejam servidores em atividade e os
carrega o bebê só viaja se e somente se for com outros dois, aposentados, então a quantidade de
ele (admita que o bebê conta como passageiro). comissões distintas que se poderá formar será
igual a:
a) 60. d) 13.
b) 30. e) 10.
EsPCEx – AFA – EFOMM – EEAR | MATEMÁTICA

c) 25.

07. (ENEM) A escrita Braile para cegos é um sistema


a) 210 c) 98 de símbolos no qual cada caráter é um conjunto
de 6 pontos dispostos em forma retangular, dos
b) 105 d) 49
quais pelo menos um se destaca em relação aos
demais. Por exemplo, a letra A é representada por
03. Um químico dispõe de cinco tubos rotulados
de A, B, C, D e E, cada um deles contendo
substâncias químicas diferentes. Ele pretende
misturar duas ou mais dessas substâncias.
Determine quantas misturas podem ser feitas, O número total de caracteres que podem ser
de modo que A e B nunca estejam juntas, pois representados no sistema Braile é:
podem causar explosão. a) 12.
a) 36 c) 25 b) 31.
b) 26 d) 18 c) 36.
d) 63.
04. Considere 8 pontos distintos em uma circunferência. e) 720.
Quantos triângulos podemos desenhar com os
vértices nesses pontos? 08. (ENEM) O código de barras, contido na maior
parte dos produtos industrializados, consiste
num conjunto de várias barras que podem estar
preenchidas com cor escura ou não. Quando um
leitor óptico passa sobre essas barras, a leitura de
uma barra clara é convertida no número 0 e a de
uma barra escura, no número 1. Observe abaixo
um exemplo simplificado de um código em um
sistema de código com 20 barras.

a) 35 d) 10
b) 25 e) 210
c) 56

05. São dadas duas retas paralelas r e s. Em r


Se o leitor óptico for passado da esquerda para
foram marcados oito pontos distintos e em s
a direita irá ler: 01011010111010110001.
seis pontos, como na figura. Quantos triângulos
podem ser formados, unindo esses pontos? Se o leitor óptico for passado da direita para a
esquerda irá ler: 10001101011101011010.
No sistema de código de barras, para se
organizar o processo de leitura óptica de cada
código, deve-se levar em consideração que
alguns códigos podem ter leitura da esquerda
para a direita igual à da direita para a esquerda,
a) 560 c) 364
como o código 00000000111100000000, no
b) 480 d) 288 sistema descrito anteriormente.
OS.:2196/13-Juliana
36
Prof. Fábio Frota

Em um sistema de códigos que utilize apenas cinco barras, a quantidade de códigos com leitura da
esquerda para a direita igual à da direita para a esquerda, desconsiderando-se todas as barras claras
ou todas as escuras, é:
a) 14
b) 12
c) 8
d) 6
e) 4

09. (ENEM) A seguir, temos parte de uma tabela de pontos, para fins de redução de peso. A dieta consiste em

EsPCEx – AFA – EFOMM – EEAR | MATEMÁTICA


a pessoa comer o que quiser, desde que não ultrapasse a quantidade diária de pontos recomendada.

Sendo assim, uma pessoa deseja fazer uma refeição contendo uma porção de carne das descritas na tabela
com até 45 pontos, comer um cereal ou farináceo com até 10 pontos e tomar uma bebida não alcoólica de
exatamente 30 pontos. Com isso, o número de opções que esta pessoa tem para escolher sua refeição é
de:
a) 52
b) 156
c) 120
d) 28
e) 5

10.
A arte do cinema
O cinema (abreviação de “cinematógrafo”, do francês cinématographe,
composto dos elementos gregos κíνημ “movimento” e  “escrever”) é a
técnica de projetar fotogramas (quadros) de forma rápida e sucessiva para criar
a impressão de movimento, bem como a arte de se produzir obras estéticas,
narrativas ou não, com esta técnica. Compreende, portanto, uma técnica, uma
forma de comunicação, uma indústria e uma arte.
Por metonímia, a palavra cinema também pode se referir ao conjunto
de pessoas que trabalham na indústria cinematográfica, ou ainda à sala de
espetáculos onde são projetadas obras cinematográficas
SITE WIKIPÉDIA - http://pt.wikipedia.org/wiki/Cinema
37 OS.:2196/13-Juliana
Prof. Fábio Frota

Determine o número de maneiras pelas quais 9 Consideremos uma urna que contenha 49
cópias de filmes podem ser distribuídas entre 4 bolas azuis e 1 bola branca. Para uma retirada,
salas de projeção, de modo que a menor sala teremos duas possibilidades: bola azul ou bola
receba 3 cópias dos filmes e cada uma das outras branca. Percebemos entretanto que será muito mais
salas receba 2 cópias dos filmes. frequente obtermos numa retirada, uma bola azul,
resultando daí, podermos afirmar que o evento “sair
a) 9340
bola azul” tem maior PROBABILIDADE de ocorrer do
b) 8820 que o evento “sair bola branca”.
c) 7560
d) 5420 Definição
Seja E um espaço amostral finito e não-vazio; e seja
EsPCEx – AFA – EFOMM – EEAR | MATEMÁTICA

A um evento desse espaço. Chama-se “probabilidade


Probabilidade – Parte I de A”, indicando-se por P(A), o número n(A)/n(E),
onde n(A) e n(E) indicam os números de elementos
Mentalize de A e E, respectivamente.

Introdução P(A) = n(A) / n(E)

Com certeza você já utilizou o conceito de


Exemplo 1
probabilidade, mesmo sem saber. Quer ver? Quantas
vezes já dissemos frases do tipo “a probabilidade Considere o lançamento de um dado. Calcule
de alguém ganhar na Mega Sena é muito pequena, a probabilidade de sair:
ele teve muita sorte” ou “a probabilidade de nós
sermos promovidos é bem grande, afinal, fizemos
um bom trabalho”. Quando falamos da porcentagem
de chance de um determinado evento ocorrer,
estamos falando de probabilidade, mas agora vamos
aprender a quantificar isso. Saiba que, em algumas
situações, a análise combinatória estudada nas aulas
anteriores será de grande importância para o cálculo
da probabilidade.
A probabilidade é a porcentagem (fração) de
a) o número 3
chance de um determinado evento ocorrer. É um
Temos E = {1, 2, 3, 4, 5, 6} ou seja n(E) = 6 e
assunto interessante para os atuais concursos, afinal
A = {3} logo n(A) = 1.
é fácil contextualizá–lo e a resposta pode ser até
intuitiva. Por exemplo, se você é uma das dez pessoas Portanto, a probabilidade procurada será igual
que estão participando de um sorteio, sua chance a p(A) = n(A)/n(E) = 1/6.
será de 10% de ganhar, ou seja, a probabilidade de b) um número par
você ganhar é de 1 para 10 (1/10 = 10/100 = 10%). Agora o evento é A = {2, 4, 6} com 3 elementos;
logo a probabilidade procurada será p(A) = 3/6
Probabilidade = 1/2 ou P(A) = 50%.
Chama-se EXPERIMENTO ALEATÓRIO àquele Isso significa dizer que a chance é de 1 para
cujo resultado é imprevisível, porém pertence cada 2 possibilidades.
necessariamente a um conjunto de resultados c) um múltiplo de 3
possíveis denominado ESPAÇO AMOSTRAL. Agora o evento A = {3, 6} com 2 elementos; logo
Qualquer subconjunto desse ESPAÇO AMOSTRAL é a probabilidade procurada será p(A) = 2/6 = 1/3.
denominado EVENTO. d) um número menor do que 3
Em oposição aos fenômenos aleatórios, existem Temos o evento A = {1, 2} com dois elementos.
os fenômenos determinísticos, que são aqueles cujos Portanto, p(A) = 2/6 = 1/3.
resultados são previsíveis, ou seja, temos certeza dos
e) múltiplo de 7
resultados a serem obtidos.
Normalmente existem diversas possibilidades Não existe nenhum múltiplo de 7 no dado,
possíveis de ocorrência de um fenômeno aleatório, portanto P = 0
sendo a medida numérica da ocorrência de f) um quadrado perfeito
cada uma dessas possibilidades, denominada Nesse caso o evento A = {1,4} com dois
PROBABILIDADE. elementos. Portanto, p(A) = 2/6 = 1/3.
OS.:2196/13-Juliana
38
Prof. Fábio Frota

Exemplo 2
Regras
Considere o lançamento de dois dados. I. Existem 6 bolas que após serem raspadas
Calcule a probabilidade de que a soma dos aparecerão um X.
resultados seja igual 8. II. O jogador deve raspar apenas uma bolinha em
cada coluna.
III. Ganha o prêmio quem encontrar um X em cada
coluna.
IV. Se for raspado mais de uma bolinha em uma
mesma coluna o cartão fica inválido.

EsPCEx – AFA – EFOMM – EEAR | MATEMÁTICA


Solução:
Observe que neste caso, o espaço amostral E
é constituído pelos pares ordenados (i,j), onde i =
número no dado 1 e j = número no dado 2. É evidente
que teremos 36 pares ordenados possíveis do tipo (i,
j) onde i = 1, 2, 3, 4, 5 ou 6, o mesmo ocorrendo com j.
As somas iguais a 8, ocorrerão nos casos: (2,6), (3,5),
(4,4), (5,3) e (6,2). Portanto, o evento “soma igual a 8”
possui 5 elementos. Logo, a probabilidade procurada
será igual a p(A) = 5/36. Sabendo que nas colunas A e B existem dois
X em cada e que nas colunas C e D apenas uma
Exemplo 3 bolinha com X em cada. Qual a probabilidade de
Um tenista participa de um torneio em que alguém ganhar nesse jogo?
lhe restam ainda no máximo 4 partidas: com X,
Solução
com Y, com X e novamente com Y, nessa ordem.
Como na coluna A temos dois X para 3
Os resultados dos jogos são independentes; a possibilidades, a probabilidade de raspar o X é
probabilidade de ele ganhar de X é igual a 1/3, e P(A) = 2/3.
a probabilidade de ganhar de Y é 1/4. Se vencer Na coluna B temos dois X para 4 bolinhas, logo
consecutivamente três dessas partidas, será P(B) = 2/4 = 1/2
considerado campeão. Determine a probabilidade Já na coluna C, temos apenas um X para 3
bolinhas, portanto P(C) = 1/3
de que isso aconteça.
Na última coluna, existe um X para 2
Solução possibilidades, logo P(D) = 1/2
Para ganhar o jogo devemos obter sucesso nos
Observe que em relação a X temos P(Ganhar) eventos A, B, C e D.
= 1/3 e P(Perder) = 2/3, já em relação a Y temos Portanto
P(Ganhar) = 1/4 e P(Perder) = 3/4. P(Ganhar) =P(A).P(B).P(C).P(C) = 2/3.1/2.1/3.1/2
Existem 3 possibilidades: = 1/18
1o) Ganhar todas as partidas P(GGGG) =
1/3.1/4.1/3.1/4 = 1/144 Modelos matemáticos
2o) Perder só a primeira (PGGG) = 2/3.1/4.1/3.1/4 = É muito importante distinguir o próprio fenômeno
e o modelo matemático para esse fenômeno.
2/144
Naturalmente, não exercemos influência sobre aquilo
3o) Perder só a última (GGGP) = 1/3.1/4.1/3.3/4 = que observamos. No entanto, ao escolher um modelo,
3/144 podemos lançar mão de nosso julgamento crítico.
Isto foi especialmente bem expresso pelo
Portanto P(Campeão) = 1/144 + 2/144 + 3/144 =
Prof. J. Neyman, que escreveu: “Todas as vezes
6/144 = 1/24 que empregarmos Matemática a fim de estudar
alguns fenômenos de observação, deveremos
Exemplo 4
essencialmente começar por construir um modelo
Temos a seguir a frente e o verso de um jogo matemático (determinístico ou probabilístico) para
de raspadinha. Leia a atentamente as regras. esses fenômenos. Inevitavelmente, o modelo deve
39 OS.:2196/13-Juliana
Prof. Fábio Frota

simplificar as coisas e certos pormenores devem empregamos a mesma espécie de considerações


ser desprezados. O bom resultado do modelo para especificar uma distribuição de probabilidade.
depende de que os pormenores desprezados sejam Estamos agora em condições de examinar
ou não realmente sem importância na elucidação e definir o que entendemos por um experimento
do fenômeno estudado. A resolução do problema “aleatório” ou “não-determinístico”.
matemático pode estar correta e, não obstante, estar
em grande discordância com os dados observados, Definição
simplesmente porque as hipóteses básicas feitas É aquele que se pode repetir infinitas vezes sob
não sejam confirmadas. Geralmente é bastante condições semelhantes e, embora não possamos
difícil afirmar com certeza se um modelo matemático precisar qual será o resultado de uma realização
especificado é ou não adequado, antes que alguns particular, podemos descrever o conjunto de todos os
EsPCEx – AFA – EFOMM – EEAR | MATEMÁTICA

dados de observação sejam obtidos. A fim de verificar seus possíveis resultados.


a validade de um modelo, deveremos deduzir um
Exemplo:
certo número de consequências de nosso modelo e,
a seguir, comparar esses resultados previstos com • E1: Jogue um dado e observe o número mostrado
observações.” na face de cima.
Deveremos nos lembrar das ideias acima • E2: Jogue uma moeda quatro vezes o observe o
enquanto estivermos estudando alguns fenômenos número de caras obtido.
de observação e modelos apropriados para sua • E3: Em uma linha de produção, fabrique peças
explicação. Vamos examinar, inicialmente, o que em série e conte o número de peças defeituosas
se pode adequadamente denominar modelo produzidas em um período de 24 horas.
determinístico. Por essa expressão pretendemos nos • E4: Um míssil récem-lançado é observado nos
referir a um modelo que estipule que as condições instantes t1, t2, . . . ,tn. Em cada um desses
sob as quais um experimento seja executado instantes, a altura do míssil acima do solo é
determinem o resultado do experimento. Por registrada.
exemplo, se introduzirmos uma bateria em um circuito
• E5: De uma urna, que só contém bolas pretas,
simples, o modelo matemático que, presumivelmente,
tira-se uma bola e verifica-se sua cor.
descreveria o fluxo de corrente elétrica observável
seria I=E/R, isto é, a Lei de Ohm.
Na natureza, existem muitos exemplos de O que os experimentos acima têm em comum? Os
“experimentos”, para os quais modelos determinísticos seguintes traços pertinentes à nossa caracterização
são apropriados. Por exemplo, as leis da gravitação de um experimento aleatório:
explicam bastante precisamente o que acontece a um Cada experimento poderá ser repetido
corpo que cai sob determinadas condições. indefinidamente sob condições essencialmente
Para um grande número de situações, o modelo inalteradas.
matemático determinístico apresentado acima Muito embora, não sejamos capazes de afirmar
é suficiente. Contudo, existem também muitos que resultado particular ocorrerá, seremos capazes
fenômenos que requerem um modelo matemático de descrever o conjunto de todos os possíveis
diferente para sua investigação. São os que resultados do experimento.
denominaremos modelos não-determinísticos ou Quando o experimento for executado
probabilísticos. (Outra expressão muito comumente repetidamente, os resultados individuais parecerão
empregada é modelo estocástico.) ocorrer de uma forma acidental. Contudo, quando o
Arriscando-nos a adiantarmos demais na experimento for repetido um grande número de vezes,
apresentação de um conceito que será definido uma configuração definida ou regularidade surgirá.
posteriormente, vamos apenas afirmar que, É esta regularidade que torna possível construir um
em um modelo determinístico, admite-se que o modelo matemático preciso, com o qual se analisará
resultado efetivo (numérico ou de outra espécie) o experimento.
seja determinado pelas condições sob as quais o
experimento ou o procedimento seja executado. Espaço amostral (S)
Em um modelo não-determinístico, no entanto, as
condições da experimentação determinam somente o Definição
comportamento probabilístico (mais especificamente, Para cada experimento aleatório definimos o
a lei probabilística) do resultado observável. ESPAÇO AMOSTRAL como conjunto de todos os
Em outras palavras, em um modelo determinístico resultados possíveis do “experimento”.
empregamos “considerações físicas” para prever Exemplo: Daremos os exemplos referentes aos
o resultado, enquanto em um modelo probabilístico “experimentos” acima:
OS.:2196/13-Juliana
40
Prof. Fábio Frota

S1 = { 1, 2, 3, 4, 5, 6 }. (1) 0  P(A)  1.
S2 = { 0, 1, 2, 3, 4 }. (2) P(S) = 1.
S3 = { 0, 1, 2,. . . ,N }, onde N é o número máximo (3) Se A e B forem eventos mutuamente
que pode ser produzido em 24h. excludentes, P(A  B)=P(A) + P(B).
S4 = { h1, h2,. . . , hn/hi ≥ 0, i= 1, 2, . . . , n }.
(4) Se A1, A2, . . . , An, . . . forem, dois a dois,
S5 = { bola preta }.
eventos mutuamente excludentes, então,
P (i=1 Ai) = P(A1) + P(A2) + . . . + P(An) + . . .
Eventos
Definição Observe-se que a Propriedade 3, decorre
É qualquer subconjunto de um “espaço amostral”. imediatamente que, para qualquer n finito,

EsPCEx – AFA – EFOMM – EEAR | MATEMÁTICA


Alguns exemplos de eventos são dados a P (∪ni=1 Ai) = ∑ P(Ai) .
seguir. Novamente, nos referimos aos experimentos Teorema 1. Se ∅ for o conjunto vazio, então
relacionados acima: Ai se referirá ao evento P(∅) = 0.
associado ao experimento Ei: Teorema 2. Se Ac for o evento complementar de
A1: Um número par ocorre, isto é, A1 = {2, 4, 6}. A, então P(A) = 1 – P(Ac).
A2: {2}; isto é, duas caras ocorrem. Teorema 3. Se A e B forem dois eventos
A3: {0}; isto é, todas as peças são perfeitas. quaisquer, então P(A ∪ B)=P(A) + P(B) – P(A ∩ B).
Teorema 4. Se A, B e C forem três eventos
Combinação de eventos quaisquer, então
Agora, poderemos empregar as várias técnicas P(A ∪ B ∪ C)=P(A) + P(B) + P(C) – P(A ∩ B) –
de combinar conjuntos (isto é, eventos) e obter novos P(A ∩ C) – P(B ∩ C) + P(A ∩ B ∩ C).
conjuntos (isto é, eventos), os quais já apresentamos Teorema 5. Se A ⊂ B, então P(A) ≤ P(B).
anteriormente.
(a) Se A e B forem eventos A ∪ B será o evento que
Resultados igualmente verossímeis
ocorrerá se, e somente se, A ou B (ou ambos)
ocorrerem. (igualmente prováveis)
(b) Se A e B forem eventos, A ∩ B será o evento que Se todos os k resultados forem igualmente
ocorrerá se, e somente se, A e B ocorrerem. verossímeis, segue-se que cada probabilidade será
pi = 1/k. Consequentemente, a condição pi, +. . . + pk = 1
(c) Se A for um evento, Ac será o evento que ocorrerá
torna-se kpi = 1 para todo i.
se, e somente se, não ocorrer A.
Essa maneira de cálculo é enunciada da seguinte
forma:
Eventos mutuamente exclusivos P(A) = n(A)/n(S).
(excludentes) Onde:
n(A) é o número de elementos ao evento A.
Definição n(S) é o número de elementos possíveis do
Dois eventos, A e B, são denominados espaço amostral S.
mutuamente excludentes, se eles não puderem ocorrer
juntos. Exprimiremos isso escrevendo A ∩ B = ∅, isto
é, a interseção de A e B é o conjunto vazio.
Exemplo. Um dispositivo eletrônico é ensaiado e Exercícios de classe
o tempo total de serviço t é registrado. Admitiremos
que o espaço amostral seja {t / t  0}. Sejam A, B e C
três eventos definidos da seguinte maneira: 01. Em uma entrevista com 100 alunos verificou-se
A = {t / t < 100}; B = {t / 50  t  200}; que 80 gostam de matemática, 60 gostam de
C = {t / t > 150}. informática e 50 gostam das duas disciplinas.
Determine a probabilidade de escolhermos um
Noções fundamentais de desses 100 alunos e ele:
probabilidade a) não gostar de nenhuma das disciplinas.
Definição Inicialmente vamos preencher o diagrama:
Seja E um experimento. Seja S um espaço
amostral associado a E. A cada evento A associaremos
um número real representado por P(A) e denominado
probabilidade de A, que satisfaça às seguintes
propriedades: Então a probabilidade é P = 10/100 = 10%
41 OS.:2196/13-Juliana
Prof. Fábio Frota

b) gostar somente de matemática.


Exercícios de casa 10
P = 30/100 = 30%

c) gostar somente de informática. 01. Num quintal existem 15 casinhas numeradas


P = 10/100 = 10% de 1 a 15 e dispostas uniformemente em torno
de um círculo. Um rato será solto no centro. A
d) gostar de matemática e informática. probabilidade de o rato entrar numa casa em que
P = 50/100 = 50% o número é múltiplo de 4 é:

a) 0,07
e) gostar de matemática ou informática.
EsPCEx – AFA – EFOMM – EEAR | MATEMÁTICA

b) 0,13
P = 90/100 = 90% c) 0,20
d) 0,25
02. Uma urna contém dez bolas numeradas de 1 a
10. Determine a probabilidade de ocorrerem os e) 0,30
seguintes casos:
a) retirar um 10. 02. (CESGRANRIO) Analisando um lote de 360 peças
P(10) = 1/10 = 10% para computador, o departamento de controle de
b) retirar um número par. qualidade de uma fábrica constatou que 40 peças
P(PAR) = 5/10 = 1/2 = 50% estavam com defeito. Retirando-se uma das 360
c) retirar um número primo. peças, ao acaso, a probabilidade de esta peça
P(PRIMO) = 4/10 = 40% NÃO ser defeituosa é:

a) 1/9
d) retirar dois números ímpares em seguida, com b) 2/9
reposição.
c) 5/9
P(II) = (5/10).(5/10) = 25/100 = 25%
d) 7/9

e) retirar três números ímpares em seguida, sem e) 8/9


reposição.
P(III) = (5/10).(4/9).(3/8) = 1/12 03. No lançamento de dois dados, qual a probabilidade
de obtermos nas faces voltadas para cima dois
03. No lançamento de moedas não viciadas, determine números ímpares?
o que se pede:
a) a probabilidade de lançar uma moeda e o
resultado ser cara.
P(K) = 1/2 = 50%

b) a probabilidade de lançar duas moedas e a) 1/4


ambas terem cara como resultado b) 1/3
P(K∩K) = P(K).P(K) = 1/2.1/2 = 1/4 = 25% c) 1/8
d) 1/2
c) a probabilidade de lançar três moedas e todas
terem cara como resultado. 04. Em um campeonato de tiro ao alvo, dois finalistas
P(K∩K∩K) = P(K).P(K).P(K) = 1/2.1/2.1/2 = atiram num alvo com probabilidade de 60% e 70%,
1/8 = 12,5% respectivamente, de acertar. Nessas condições, a
probabilidade de ambos errarem o alvo é:

d) a probabilidade de lançar três moedas e pelo a) 30%


menos uma ter cara como resultado. b) 42%
P = 1 – P(K∩K∩K) = 1 – P(K).P(K).P(K) = 1 – c) 50%
1/2.1/2.1/2 = 1 – 1/8 = 100% – 12,5% = 87,5% d) 12%

OS.:2196/13-Juliana
42
Prof. Fábio Frota

05. Sabendo que um atirador tem 70% de chance 11. No lançamento de um dado viciado, a probabilidade
de acertar um tiro no centro do alvo, determine a de cair o número 6 é de 20% e o restante dos
probabilidade dele atirar duas vezes seguidas e números tem igual probabilidade. Determine a
acertar pelo menos uma das vezes. probabilidade de jogar esse dado e o resultado
ser par.
a) 62%
b) 52%
c) 48%
d) 36%
e) 16%

EsPCEx – AFA – EFOMM – EEAR | MATEMÁTICA


12. No lançamento de um dado viciado, a probabilidade
a) 9% de sair o número 6 é de 30%, de sair o número
b) 49% 1 é de 10% e o restante dos números tem igual
probabilidade. Qual a probabilidade de jogar um
c) 51%
dado duas vezes seguidas e o resultado ser par
d) 91%
nos dois lançamentos?
a) 24%
06. Ao lançar três dados qual a probabilidade do
produto dos resultados não ser par? b) 36%
c) 40%
a) 1/8
d) 60%
b) 3/8
e) 64%
c) 5/8
d) 7/8

Probabilidade – Parte II
07. Qual a probabilidade de lançar dois dados e o
produto dos resultados ser um número par?
Probabilidade condicional
a) 1/2 Na Tabela 1 temos dados referentes a alunos
b) 1/4 matriculados em quatro cursos de uma universidade
c) 3/4 em dado ano.
d) 3/5
Tabela 1: Distribuição de alunos segundo o sexo e
escolha de curso
08. Um casal pretende ter 3 filhos, qual a probabilidade
de pelo menos um deles ser homem? Sexo
Curso Homens Mulheres
a) 1/8 Total
(H) (M)
b) 7/8
Matemática Pura (M) 70 40 110
c) 5/8
Matemática Aplicada
d) 3/8 15 15 30
(A)
Estatística (E) 10 20 30
09. No lançamento de 6 moedas não viciadas, qual
Sexo
a probabilidade de pelo menos uma das moedas
Curso Homens Mulheres
ser cara? Total
(H) (M)
a) 1/64 c) 6/64 Computação (C) 20 10 30
b) 63/64 d) 31/32 Total 115 85 200

10. Jogando um dado, não viciado, qual a probabilidade Dado que um estudante, escolhido ao acaso,
de tirarmos um número maior que 4 duas vezes esteja matriculado no curso de Estatística, a
seguidas? probabilidade de que seja mulher é 20/30 = 2/3. Isso
a) 1/3 c) 1/8 porque, do total de 30 alunos que estudam Estatística,
20 são mulheres. Escrevemos
b) 1/6 d) 1/9
P(mulher/Estatística) = 2/3.
43 OS.:2196/13-Juliana
Prof. Fábio Frota

Para dois eventos quaisquer A e B, sendo P(B) > 0, i) Três cartas em seguida, com reposição, e pelo
definimos a probabilidade condicional de A dado B, menos uma delas ser de ouro.
P(A/B), como sendo Como devemos tirar três cartas e pelo menos
P(A/B) = P(A  B)/P(B) . uma tem que ser ouro, concluímos que a única
Observe que P(A) = P(mulher) = 85/200 = 17/40, coisa que não pode ocorrer é tirar três cartas
e com a informação de que B ocorreu ( o aluno é seguidas que não sejam de ouro, então a
matriculado em Estatística), obtemos P(A/B) = 2/3. probabilidade procurada é
Podemos dizer que P(A) é a probabilidade a priori de P = 1 – (3/4).(3/4).(3/4) = 1 – 27/64 = 37/64
A e, com a informação adicional de que B ocorreu,
j) Um rei (K), dado que a carta é de ouro.
obtemos a probabilidade a posteriori P(A/B). Note
Entre as 13 cartas de ouro, existe apenas um
que, nesse caso, P(A/B) > P(A), logo a informação de
EsPCEx – AFA – EFOMM – EEAR | MATEMÁTICA

rei (K), logo


que B ocorreu aumentou a chance de A ocorrer.
P(K/♦) = P(K∩♦)/P(♦) = 1/13
k) Uma carta de ouro, dado que a carta retirada
Exercícios resolvidos é um rei (K).
01. De um baralho de 52 cartas (13 de cada naipe: Entre os 4 reis do baralho, apenas uma carta
♣, ♠, ♦ ou ♥), determine a probabilidade de ser é de ouro, logo
retirada: P(♦/K) = P(♦∩K)/P(K) = 1/4
a) Um ás (A).
P(A) = 4/52 = 1/13 02. Em uma sala com 50 alunos, 28% deles usam
b) Uma carta de ouro. óculos, 40% são homens e 60% dos homens
P(♦) = 13/52 = 1/4 = 25% não usam óculos.
c) Um ás (A) de ouro. Determine a probabilidade de sortear:
Como a distribuição das cartas é uniforme,
temos
P(A ∩ ♦) = P(A) . P(♦) = 1/13 . 1/4 = 1/52
De outra forma, podemos simplesmente ver
que só existe um As de ouro, dentre as 52
cartas, logo
P(A ∩ ♦) = 1/52
d) Um ás (A) ou uma carta de ouro.
P(A ∪ ♦) = P(A) + P(♦) – P(A ∩ ♦)
P(A ∪ ♦) = 4/52 + 13/52 – 1/52 = 16/52
P(A ∪ ♦) = 4/13 DADOS NA TABELA
e) Uma carta com figura (J, Q ou K).
H M
Existem 4 valetes (J), 4 damas (Q) e 4 reis (K),
O 8 6 14
logo
~O 12 24 36
P(J ∪ Q ∪ K) = 12/52 = 3/13
20 30 50
f) Três reis em seguida, sem reposição.
Como as cartas retiradas não vão sendo
a) uma mulher
devolvidas, a probabilidade de retirar o
próximo rei vai diminuindo, ou seja, P(M) = 30/50 = 3/5 = 60%
P(K∩K∩K) = (4/52).(3/51).(2/50) = 1/5525 b) uma pessoa de óculos
g) Uma carta que não seja de ouro. P(O) = 14/50 = 7/25 = 28%
A chance de tirar uma carta de ouro é P(♦) = c) uma mulher de óculos
1/4 e de não tirar é P(♦) = 1 – P(♦), ou seja P(M∩O) = 6/50 = 3/25 = 12%
P(♦) = 3/4
h) Três cartas em seguida, com reposição, e d) uma mulher ou uma pessoa que esteja de
todas não serem de ouro. óculos
Como há reposição, a probabilidade de retirar P(M∪O) = (8+6+24)/50 = 38/50 = 19/25 = 76%
uma carta que não seja de ouro é sempre a e) uma mulher, dado que ela está de óculos
mesma, logo P(M/O) = 6/14 = 3/7 (ser mulheres dentre
P(♦∩♦∩♦) = (3/4).(3/4).(3/4) = 27/64 aqueles que estão de óculos)
OS.:2196/13-Juliana
44
Prof. Fábio Frota

f) uma pessoa de óculos, dado que ela é uma 03. (ENEM) Um município de 628 km² é atendido
mulher por duas emissoras de rádio cujas antenas A e B
P(O/M) = 6/30 = 1/5 = 20% (está de óculos alcançam um raio de 10km do município, conforme
dentre as mulheres) mostra a figura:

Exercícios de casa 11
01. (ENEM) As 23 ex-alunas de uma turma que
completou o Ensino Médio há 10 anos se
encontraram em uma reunião comemorativa.

EsPCEx – AFA – EFOMM – EEAR | MATEMÁTICA


Várias delas haviam se casado e tido filhos.
A distribuição das mulheres, de acordo com a
quantidade de filhos, é mostrada no gráfico abaixo. Para orçar um contrato publicitário, uma agência
precisa avaliar a probabilidade que um morador
tem de, circulando livremente pelo município,
encontrar-se na área de alcance de pelo menos
uma das emissoras.
Essa probabilidade é de, aproximadamente,
a) 20%.
b) 25%.
c) 30%.
Um prêmio foi sorteado entre todos os filhos
d) 35%.
dessas ex-alunas. A probabilidade de que a
criança premiada tenha sido um(a) filho(a) e) 40%.
único(a) é:
a) 1/3. d) 7/23. 04. (ENEM) Num determinado bairro há duas
empresas de ônibus, ANDABEM e BOMPASSEIO,
b) 1/4. e) 7/25.
que fazem o trajeto levando e trazendo passageiros
c) 7/15.
do subúrbio ao centro da cidade. Um ônibus de
cada uma dessas empresas parte do terminal a
02. (ENEM) Uma empresa de alimentos imprimiu cada 30 minutos, nos horários indicados na tabela.
em suas embalagens um cartão de apostas do
seguinte tipo: Horário dos ônibus
Andabem Bompasseio
… ...
6h00min 6h10min
6h30min 6h40min
7h00min 7h10min
7h30min 7h40min
… …

Cada cartão de apostas possui 7 figuras de bolas


Carlos mora próximo ao terminal de ônibus e
de futebol e 8 sinais de “X” distribuídos entre
trabalha na cidade. Como não tem hora certa para
os 15 espaços possíveis, de tal forma que a
chegar ao trabalho e nem preferência por qualquer
probabilidade de um cliente ganhar o prêmio nunca
das empresas, toma sempre o primeiro ônibus que
seja igual a zero.
sai do terminal. Nessa situação, pode-se afirmar
Em determinado cartão existem duas bolas na que a probabilidade de Carlos viajar num ônibus
linha 4 e duas bolas na linha 5. Com esse cartão, da empresa ANDABEM é:
a probabilidade de o cliente ganhar o prêmio é:
a) um quarto da probabilidade de ele viajar num
a) 1/27. d) 1/72. ônibus da empresa BOMPASSEIO.
b) 1/36. e) 1/108. b) um terço da probabilidade de ele viajar num
c) 1/54. ônibus da empresa BOMPASSEIO.
45 OS.:2196/13-Juliana
Prof. Fábio Frota

c) metade da probabilidade de ele viajar num 07. (ENEM) Em um concurso de televisão, apresentam-
ônibus da empresa BOMPASSEIO. se ao participante 3 fichas voltadas para baixo,
estando representada em cada uma delas as letras
d) duas vezes maior do que a probabilidade de ele
T, V e E. As fichas encontram-se alinhadas em uma
viajar num ônibus da empresa BOMPASSEIO.
ordem qualquer. O participante deve ordenar as
e) três vezes maior do que a probabilidade de ele fichas ao seu gosto, mantendo as letras voltadas
viajar num ônibus da empresa BOMPASSEIO. para baixo, tentando obter a sigla TVE. Ao desvirá-
las, para cada letra que esteja na posição correta
ganhará um prêmio de R$ 200,00.
05. (ENEM) Um apostador tem três opções para
participar de certa modalidade de jogo, que A probabilidade de o participante não ganhar
qualquer prêmio é igual a:
EsPCEx – AFA – EFOMM – EEAR | MATEMÁTICA

consiste no sorteio aleatório de um número dentre


dez. a) 0 d) 1/2
1a opção: comprar três números para um único b) 1/3 e) 1/6
sorteio. c) 1/4

2a opção: comprar dois números para um sorteio


08. (ENEM) Em um concurso de televisão, apresentam-
e um número para um segundo sorteio. se ao participante 3 fichas voltadas para baixo,
3a opção: comprar um número para cada sorteio, estando representada em cada uma delas as letras
num total de três sorteios. T, V e E. As fichas encontram-se alinhadas em uma
ordem qualquer. O participante deve ordenar as
Se X, Y, Z representam as probabilidades de o fichas ao seu gosto, mantendo as letras voltadas
apostador ganhar algum prêmio, escolhendo, para baixo, tentando obter a sigla TVE. Ao desvirá-
respectivamente, a 1a, a 2a ou a 3a opções, é las, para cada letra que esteja na posição correta
correto afirmar que: ganhará um prêmio de R$ 200,00.

a) X < Y < Z. A probabilidade de o concorrente ganhar


exatamente o valor de R$ 400,00 é igual a:
b) X = Y = Z.
a) 0
c) X >Y = Z.
b) 1/3
d) X = Y > Z.
c) 1/2
e) X > Y > Z. d) 2/3
e) 1/6
06. (ENEM) Um apostador tem três opções para
participar de certa modalidade de jogo, que 09. (ENEM) Uma pesquisa de opinião foi realizada
consiste no sorteio aleatório de um número dentre para avaliar os níveis de audiência de alguns
dez. canais de televisão, entre 20h e 21h, durante uma
determinada noite. Os resultados obtidos estão
1a opção: comprar três números para um único representados no gráfico de barras abaixo:
sorteio.

2a opção: comprar dois números para um sorteio


e um número para um segundo sorteio.

3a opção: comprar um número para cada sorteio,


num total de três sorteios.

Escolhendo a 2a opção, a probabilidade de o


apostador não ganhar em qualquer dos sorteios
é igual a:

a) 90%. O número de residências atingidas nessa


b) 81%. pesquisa foi aproximadamente de:
c) 72%. a) 100 d) 200
d) 70%. b) 135 e) 220
e) 65%. c) 150

OS.:2196/13-Juliana
46
Prof. Fábio Frota

10. (ENEM) A percentagem de entrevistados necessidades da agricultura. Na representação a


que declararam estar assistindo à TvB é seguir, o sexto mês do ano, você pode observar
aproximadamente igual a: o dia de hoje, 6/6/2009, em que você avalia seus
conhecimentos.
a) 15%
b) 20%
c) 22%
d) 25%
e) 30%

11. (ENEM) Escolhendo-se aleatoriamente um

EsPCEx – AFA – EFOMM – EEAR | MATEMÁTICA


dos entrevistados, a probabilidade dele estar
assistindo à TvB ou TvC é aproximadamente Sabendo que um dia desse mês, que não o
igual a: dia de Corpus Christi, foi sorteado ao acaso, a
a) 50% d) 25% probabilidade de ter sido sorteado um dia ímpar
ou múltiplo de seis é:
b) 20% e) 30%
c) 22% a) 2/3 d) 1/12
b) 19/29 e) 30/29
12. (ENEM) Uma das principais causas da c) 7/90
degradação de peixes frescos é a contaminação 14. Carlos diariamente almoça um prato de sopa
por bactérias. O gráfico apresenta resultados no mesmo restaurante. A sopa é feita de forma
de um estudo acerca da temperatura de peixes aleatória por um dos três cozinheiros que lá
frescos vendidos em cinco peixarias. trabalham:
• 40% das vezes a sopa é feita por João;
• 40% das vezes por José;
• 20% das vezes por Maria;
• João salga demais a sopa 10% das vezes;
• José o faz em 5% das vezes;
• Maria 20% das vezes.
Como de costume, um dia qualquer Carlos pede
a sopa e, ao experimentá-la, verifica que está
salgada demais. A probabilidade de que essa
sopa tenha sido feita por José é igual a:
O ideal é que esses peixes sejam vendidos com a) 0,15. d) 0,20.
temperaturas entre 2 ºC e 4 ºC. Selecionando-
b) 0,25. e) 0,40.
-se aleatoriamente uma das cinco peixarias
c) 0,30.
pesquisadas, a probabilidade de ela vender
peixes frescos na condição ideal é igual a: 15. Marco estuda em uma universidade na qual,
a) 1/2 d) 1/5 entre as moças de cabelos loiros, 18 possuem
b) 1/3 e) 1/6 olhos azuis e 8 possuem olhos castanhos;
entre as moças de cabelos pretos, 9 possuem
c) 1/4
olhos azuis e 9 possuem olhos castanhos;
entre as moças de cabelos ruivos, 4 possuem
13. (ENEM) Os calendários se baseiam nos olhos azuis e 2 possuem olhos castanhos.
movimentos aparentes dos dois astros mais Marisa seleciona aleatoriamente uma dessas
brilhantes da cúpula celeste, o Sol e a Lua, para moças para apresentar para seu amigo Marco.
determinar as unidades de tempo: dia, mês Ao encontrar com Marco, Marisa informa que
e ano. O dia, cuja noção nasceu do contraste a moça selecionada possui olhos castanhos.
entre a luz solar e a escuridão da noite, é o Com essa informação, Marco conclui que a
elemento mais antigo e principal do calendário. A probabilidade de a moça possuir cabelos loiros
observação da periodicidade das fases lunares ou ruivos é igual a:
gerou a ideia de mês, e a repetição alternada a) 0 d) 10/50
das estações, de acordo com os climas, deu b) 10/19 e) 19/31
origem ao conceito de ano, criado em função das c) 19/50
47 OS.:2196/13-Juliana
Prof. Fábio Frota

Número binomial e Pascal


Blaise Pascal (1623-1662)
Filósofo e Matemático francês, nasceu em
Clermont e morreu em 1662 na cidade de Paris.
O pai de Pascal, que tinha uma concepção
educacional pouco ortodoxa, decidiu que seria
ele próprio a ensinar os filhos e que Blaise não
estudaria Matemática antes dos 15 anos, pelo que
EsPCEx – AFA – EFOMM – EEAR | MATEMÁTICA

mandou remover de casa todos os livros e textos


matemáticos. Contudo, movido pela curiosidade
Blaise começou a trabalhar em Geometria a partir
dos 12 anos, chegando mesmo a descobrir, por si,
que a soma dos ângulos de um triângulo é igual
a dois ângulos retos(180 o). Aos dezoito anos e Abaixo está representado o triângulo de Pascal
com os valores já calculados.
com o objetivo de ajudar o pai na tarefa de cobrar
1
impostos, Pascal inventou a primeira máquina 1 1
digital, chamada Pascalinne para levar a cabo o 1 2 1
processo de adição e subtração e posteriormente 1 3 3 1
1 4 6 4 1
organizou a produção e comercialização destas
1 5 10 10 5 1
máquinas de calcular (que se assemelhava a uma 
calculadora mecânica dos anos 40). Pelo menos
Observação
sete destes “computadores” ainda existem.
Se somarmos os elementos da linha n obteremos
Introdução 2n.
Anteriormente, vimos a combinação de n 1 = 20
elementos tomados p a p, indicada pelo símbolo Cn,p. 1 + 1 = 21
1 + 2 + 1 = 22
n
Agora, vamos indicar também pelo símbolo   , 1 + 3 + 3 + 1 = 23
p 1 + 4 + 6 + 4 + 1 = 24
chamando de número binomial de n sobre p.

n n!
  Para {n,p} ⊂ N, p  n
p
  p !.(n  p)!

Propriedades
Ex1.: 1. Números binomiais complementares são
iguais
5 5! 5! 5 .4 .3 !
     10
 3  3 !.(5  3)! 3 !.2! 3 !.2.1 n  n 
  
p n  p

Triângulo de Pascal Ex2.:

 7   7   10   10  6 6
Pascal criou uma tabela de números binomiais      ,      ou     
 2 5  4   6  0 6
n
onde cada número   está localizado na linha n e
p 2. Sendo n, p e q números naturais com n  p e n
coluna p, como mostrado a seguir. Vamos em seguida  q.
observar uma série de propriedades curiosas que n n p  q ou
podemos destacar.    
p q
    p  q  n

OS.:2196/13-Juliana
48
Prof. Fábio Frota

Ex3.:
6 6 7
 10   10   x  3 ou 02. Qual o valor da soma         ?
    3 4 5
x 3  x  3  10  x  7
Ex4.: a) 65 d) 96
 15   15  m  3  12  m  15 ou b) 56 e) 07
   c) 84
 m  3   12  m  3  12  15  m  6

3. Relação de Stiffel
03. Determine a soma dos valores de x que satisfazem
 n   n   n  1 8 8 9
  

EsPCEx – AFA – EFOMM – EEAR | MATEMÁTICA


 a equação         .
 p   p  1  p  1 3 4  x
Ex5.:
a) 4 d) 7
4 4 5 b) 5 e) 6
    
 1  2   2 c) 9
Ex6.:
8 8 9 9 9  x  4 ou 04. Resolva a equação
           
3  4  x  4  x x  4  9  x  5 8 8 8 8 8
2x =          ...       .
 0   1  2  7 8
Observação
a) 2 d) 10
Saiba que frequentemente você usará a b) 4 e) 100
relação de Stifel. Essa relação se torna mais fácil c) 8
de entender olhando–a no triângulo de Pascal.
Veja que a soma de números binomiais vizinhos
n  n  Exercícios de casa 12
   é igual ao termo da linha seguinte n+1
 p   p  1
e da coluna p+1, logo abaixo portanto. 01. Dadas as afirmações a seguir:

n n n  n  n n


I.          ...      2 , n  N
0 1
      2  n  1 n
  
n  n 
II.    n ∈ N, k = 0, 1, 2, ..., n
k  n  k 

 10   11  11
III.        
5 5 6

Conclui-se que:
a) todas são verdadeiras.
b) apenas (I) e (II) são verdadeiras.
c) apenas (I) é verdadeira.
Exercícios de classe d) apenas (II) é verdadeira.
01. Quais os valores de k que tornam iguais os e) apenas (II) e (III) são verdadeiras.
 10   10 
números binomiais  e ?
 k  2   2k  4   99   99 
02. Pode–se afirmar que o valor de      é
 98   99 
a) apenas 4 e 6
b) somente o 4 igual a:
c) somente o 6 a) 1 d) 9900
d) qualquer valor de k inteiro b) 99 e) 9702
e) não existem valores que satisfaçam c) 100
49 OS.:2196/13-Juliana
Prof. Fábio Frota

03. Sejam k1 e k2 os valores de k que tornam iguais 08. Determine o valor de k que satisfaz a equação
 18   18   k  1  k  1
os números binomiais  e  , então     
 k  1  3k  5   2   3   1.
k1.k2 é igual a:  k  2
 
 5 
a) 2 d) 24
b) 6 e) zero a) 6
c) 12 b) 5
c) 4
04. Determine os valores reais de p que satisfazem a d) 3
EsPCEx – AFA – EFOMM – EEAR | MATEMÁTICA

 19   19   20  e) 2
equação         .
 14   15   p 
09. O número natural x que é solução da equação
a) 14 e 15 d) 5 e 14
 2 3  4 5
b) 5 e 15 e) apenas 15 x             é:
 0   1  2   3 
c) 2 e 3
a) primo
05. Encontre o valor de x que satisfaça a igualdade b) divisível por 2
x x x x c) múltiplo de 3
         ...     128 é: d) quadrado perfeito
0 1
      2 x
e) maior que 20
a) 6 d) 9
b) 7 e) 10
 100   100   100   100   100 
c) 8 10. A soma      ...    
 0   1   2   99   100 
é igual a 2m, então o valor de m é igual a:
06. P o d e m o s a f i r m a r q u e a soma
 20   20   21  22  a) 1
           é igual a: b) 10
 10   11   12   13 
c) 100
 23  d) 1002
a)  
 13  e) 100100

 23 
b)   Binômio de Newton
 14 

 22  Introdução
c)  
 13  Para resolvermos certos problemas que envolvam
 20  potências do tipo (x + y)n, onde x e y são números
d)  13  quaisquer e n é um número natural, é necessária a
 
utilização do teorema do binômio de Newton para
 22  facilitar o desenvolvimento.
e)  
 10  Quando o valor de n é pequeno fica fácil
compreender, observe:
( x + y )0 = 1
 n  2
07. Dada a equação    10 . Calcule n!. ( x + y )1 = x + y
 n 
( x + y )2 = x2 + 2xy + y2
a) 120
( x + y )3 = x3 + 3x2y + 3xy2 + y3
b) 24
c) 6 ( x + y )4 = x4 + 4x3y + 6x2y2 + 4xy3 + y4
d) 2 ( x + y )5 = x5 + 5x4y + 10x3y2 + 10x2y3 + 5xy4 + y5
e) 1
OS.:2196/13-Juliana
50
Prof. Fábio Frota

Observação

É interessante perceber que existe uma lógica de formação, os expoentes de x começam elevados a
n e vão decrescendo até zero, enquanto os de y começam com zero e vão crescendo até n, além disso os
coeficientes são exatamente os valores do triângulo de Pascal correspondentes da linha n.

Teorema do binômio de Newton


O matemático, físico e astrônomo inglês sir Isaac Newton demonstrou que:

n n n n


( x  y )n    .xn .y 0    .xn1.y1    .xn2 .y 2  ...    .x 0 .yn

EsPCEx – AFA – EFOMM – EEAR | MATEMÁTICA


0
  1
  2
  n

Ex1.:
Desenvolver a potência (2a + b)4.
Solução:
 4 4  4  4  4
(2a  b)4    .(2a)4 .b0    .(2a)3 .b1    .(2a)2 .y 2    .(2a)1.b3    .(2a)0 .b4
0  1  2 3  4

Os coeficientes binomiais formam a linha 4 do triângulo de Pascal.

1
1 1
1 2 1
1 3 3 1
1 4 6 4 1
1 5 10 10 5 1

logo temos (2a + b)4 = 1.16a4.1 + 4.8a3.b + 6.4a2.b2 + 4.2a.b3 + 1.1.b4
portanto (2a + b)4 = 16a4 + 32a3b + 24a2b2 + 8ab3 + b4

Somatório
Para uma melhor compreensão do assunto devemos conhecer o símbolo Σ (letra grega maiúscula
denominada “sigma”) que é utilizada para indicar um somatório.

Ex2.:
4
 (2n ) = 21 + 22 + 23 + 24 = 2 + 4 + 8 + 16 = 30
n1

Observe que nós somamos todos os valores de 2n variando n de 1 a 4, sendo n um número inteiro.

Ex3.:

= 1 + 4 + 6 + 4 + 1 = 16

Lembre-se de que essa soma também pode ser dada simplesmente por 24 = 16.

Termo geral do binômio de Newton


n
n
Vimos que (x + y)n =   p .xnp .yp
p 0  

51 OS.:2196/13-Juliana
Prof. Fábio Frota

Chamamos então de termo geral do binômio a expressão

n
Tp 1    xnp .yp
p

Ex4.:
Qual o 60 termo no desenvolvimento do binômio (x3 + 2)8 ?
Solução:
Quando a questão pedir diretamente um termo e não todo o desenvolvimento do binômio é conveniente
utilizar a fórmula do termo geral, portanto para o 60 termo (T6) teremos p=5 e n=8, logo
EsPCEx – AFA – EFOMM – EEAR | MATEMÁTICA

8
T6    ( x 3 )8 5 .25 = 56.x9. 32 = 1792
5
8 8! 8 .7 .6 . 5 !
Lembre-se de que      56
 5  5 !.3 ! 5 !.3.2.1

Ex5.:
Qual o termo independente no desenvolvimento do binômio (x2 + 2/x)12 ?
Solução:
Essa é uma questão clássica no que diz respeito a binômio de Newton, portanto fique atento.
O termo independente é aquele em que não aparece o x, ou seja, não dependente da variável.
Sendo o termo geral
p
 12  2  12  2p
Tp 1    ( x 2 )12p .      x 24 2p . p devemos encontrar um valor de p que faça com que a variável
p x p x

x desapareça. Portanto, a variável do numerador deve cancelar com a do denominador, ou seja


(x2)12-p = xp ⇒ x24-2p = xp ⇒ 24-2p = p ⇒ 3p = 24 ⇒ p = 8
logo o 90 termo (p = 8) é independente de x
8
 12  2 12! 8 28
T9    ( x 2 )128 .    .x . 8 = 495.256 = 126720
8 x 8 !.4 ! x

Ex6.: 8
 1 
Qual o termo independente de x na expansão de  5 x +  ?
3
 x

Solução:
Da mesma forma que na questão anterior, mas agora tomando o cuidado de escrever as raízes na forma
n
de potência, o termo independente é aquele em que não aparece o x. Lembre−se que xm = xm / n .
8 p
 1/ 5 1   8  1 / 5 8 p  1   8  ( 8 p ) / 5 1
T 
Dado binômio  x  1/ 3  , então o termo geral é p 1   ( x ) .  1/ 3    p  x . p/3
 x  p
  x    x

Para encontrar o valor de p que torna o termo independente de x, o numerador deve cancelar com a do
denominador, ou seja
X(8-p)/5 = xp/3 ⇒ (8−p)/5 = p/3 ⇒ 3(8−p) = 5p ⇒ 24 − 3p = 5p ⇒ 8p = 24 ⇒ p = 3
logo o 40 termo (p = 3) é independente de x
3
8  1  8 ! 1 1 8 .7 .6 .5 !
T4    ( x1/ 5 )8 3 .  1/ 3   .x . 1  = 56
3 x  3 !.5 ! x 3 .2 .1 .5 !

OS.:2196/13-Juliana
52
Prof. Fábio Frota

Exercícios de classe 03. Qual o termo independente da variável x no


15
 2 
desenvolvimento do binômio  x  4  ?
 x 
01. Dado o binômio (x2 + 1/x)12, obtenha o que se
pede: a) 3640
b) 1820
a) O 4º termo
c) 910
b) O termo médio
d) 455
c) O termo independente
e) 120
d) O coeficiente de x15

EsPCEx – AFA – EFOMM – EEAR | MATEMÁTICA


04. Se a soma dos coeficientes do desenvolvimento do
6
 6  6 p binômio (2x + y)n é igual a 243, então o número n é:
02. Determine o valor de    (8) .(2) .
p

p 0  p 
a) 12
a) 106 b) 10
b) 86 c) 8
c) 68 d) 5
d) 26 e) 3
e) 210
05. O coeficiente de x 6 no desenvolvimento de
03. D e t e r m i n e a s o m a d o s c o e f i c i e n t e s d o ( 2x 2 + 2)5 é:
desenvolvimento do binômio (5x5 – 3y9)8. a) 40 2
a) 16 b) 48 2
b) 64 c) 60 2
c) 128 d) 80 2
d) 256
e) 512 06. Considere o desenvolvimento do binômio
(2x+1/2)10 segundo as potências decrescentes de
04. No desenvolvimento do binômio (x + a)8 na x. A razão entre os coeficientes do terceiro e do
variável x, onde a é um número inteiro e positivo, quinto termos, nessa ordem, é igual a:
o 4º termo é 448x5. Determine o valor de a. a) 20/11
a) 1 b) 21/10
b) 2 c) 22/9
c) 3 d) 23/8
d) 4 e) 24/7
e) 5
07. Se o termo médio do desenvolvimento do binômio
(4x + ky)10 é 8064x5y5, então k é igual a:
Exercícios de casa 13 a) 1/4
b) 1/2
01. A soma de todos os coeficientes do desenvolvimento c) 1
de (10x3 − 9y)30 é:
d) 2
a) 0 d) 7 e) 4
b) 1 e) 13
c) –1 08. Determine o valor da expressão 185 – 5.184.8 +
10.183.82 – 10.182.83 + 5.18.84 – 85.
02. No desenvolvimento do binômio (2a3 + b/2)10, a) 1000
qual seu termo médio? b) 10000
a) 32a3b d) 252a15b5 c) 100000
b) 100a5b15 e) 8064a15b5 d) 1000000
c) 126a5b15 e) 10000000
53 OS.:2196/13-Juliana
Prof. Fábio Frota

09. O termo independente de x no desenvolvimento Exercícios de casa – 5


de (x+1/x)6 é: 01 02 03 04 05
a) 10. d) 16. c b d d a
b) 30. e) 20. 06 07 08 09 10
c) 40. b * * c 22
11 12 13 14 15
10. Considere o desenvolvimento do binômio
b – c e d
(2x+1/2)10 segundo as potências decrescentes de
x. A razão entre os coeficientes do terceiro e do 16 17 18 19 20
quinto termos, nessa ordem, é igual a: b d d b a
EsPCEx – AFA – EFOMM – EEAR | MATEMÁTICA

a) 20/11
b) 21/10 07. 12,6
c) 22/9 08. x = log89
2 ou x = 0

d) 23/8
e) 24/7
Exercícios de casa – 6
01 02 03 04 05
c c b d d
Gabaritos 06 07 08 09 10
a c b a a
Exercícios de casa – 1
01 02 03 04
d – e c Exercícios de casa – 7
05 06 01 02 03 04 05
d c d c c b a
06 07 08 09 10

Exercícios de casa – 2 d c d d b

01 02 03 04 05
– a c e b
Exercícios de casa – 8
06 07 08 09 10
01 02 03 04 05
b c c c a
c b c e d
11 12 13 14 15
06 07 08 09 10
d d 21 a c
d b b b c

Exercícios de casa – 3
01 02 03 04 05 Exercícios de casa – 9
d a e b a 01 02 03 04 05
06 07 d c d c d
b e 06 07 08 09 10
b d d a c

Exercícios de casa – 4
01 02 03 04 05 Exercícios de casa – 10
b d d c a
01 02 03 04 05 06 07 08
06 07 08 09 10
– – – – c e a d
c a a c b
09 10 11 12 13 14 15
11 12 13
d a c b b b b
c c c

OS.:2196/13-Juliana
54
Prof. Fábio Frota

Exercícios de casa – 11
01 02 03 04 05 06
e c b a d e
07 08 09 10 11 12
d d b a d b

Exercícios de casa – 12
01 02 03 04 05

EsPCEx – AFA – EFOMM – EEAR | MATEMÁTICA


b c c b b
06 07 08 09 10
a c a b c

Exercícios de casa – 13
01 02 03 04 05
b d a d d
06 07 08 09 10
e c c e e

55 OS.:2196/13-Juliana

Você também pode gostar